15

You might also like

Download as pdf or txt
Download as pdf or txt
You are on page 1of 36

https://www.pdfnotes.

co/

CO
S.
TE
NO
DF
.P
W
W
W
https://www.pdfnotes.co/

DO NOT OPEN THIS TEST BOOKLET UNTIL YOU ARE ASKED TO DO SO

PPP-PTS-4341/092023/15

GENERAL STUDIES

ENVIRONMENT - 4
Time Allowed: One Hour Maximum Marks: 100

INSTRUCTIONS

1. IMMEDIATELY AFTER THE COMMENCEMENT OF THE EXAMINATION, YOU SHOULD CHECK THAT
THIS TEST BOOKLET DOES NOT HAVE ANY UNPRINTED OR TORN OR MISSING PAGES OR ITEMS,
ETC. IF SO, GET IT REPLACED BY A COMPLETE TEST BOOKLET.
2. Please note that it is the candidate’s responsibility to encode and fill in the Roll Number carefully
without any omission or discrepancy at the appropriate places in the OMR Answer Sheet. Any
omission/discrepancy will render the Answer Sheet liable for rejection.
3. You have to enter your Roll Number on the test booklet in the Box provided
alongside. DO NOT write anything else on the Test Booklet.
4. This Test Booklet contains 50 items (questions). Each item is printed in English. Each item comprises four
responses (answers). You will select the response which you want to mark on the Answer Sheet. In case you
feel that there is more than one correct response, mark the response which you consider the best. In any case,
choose ONLY ONE response for each item.
5. You have to mark all your responses ONLY on the separate Answer Sheet provided. See directions in the
Answer Sheet.
6. All items carry equal marks.
7. Before you proceed to mark in the Answer Sheet in response to various items in the Test Booklet, you have to
fill in some particulars in the Answer Sheet as per instructions sent to you with your Admission Certificate.
8. After you have completed filling in all your responses on the Answer Sheet and the examination has concluded,
you should hand over to the Invigilator only the Answer Sheet. You are permitted to take away with you the
Test Booklet.
CO
9. Sheets for rough work are appended in the Test Booklet at the end.
10. Penalty for wrong answers:
S.

THERE WILL BE PENALTY FOR WRONG ANSWERS MARKED BY A CANDIDATE IN THE OBJECTIVE
TE

TYPE QUESTION PAPERS.


(i) There are four alternatives for the answer to every question. For each question for which a wrong answer
NO

has been given by the candidate, one-third of the marks assigned to that question will be deducted as
penalty.
(ii) If a candidate gives more than one answer, it will be treated as a wrong answer even if one of the given
DF

answers happens to be correct and there will be same penalty as above to that question.
(iii) If a question is left blank, i.e., no answer is given by the candidate, there will be no penalty for that
.P

question.
W

DO NOT OPEN THIS TEST BOOKLET UNTIL YOU ARE ASKED TO DO SO


W
W
https://www.pdfnotes.co/

1. Consider the following: Select the correct option using the code given
below:
1. Silicon dioxide (SiO2)
(a) 2 and 3 only
2. Calcium oxide (CaO)
3. Aluminium oxide (Al2O3) (b) 1 and 2 only

How many of the above chemicals are primarily (c) 1 and 3 only
present in the fly ash? (d) 1, 2 and 3
(a) Only one
(b) Only two 5. Consider the following statements with respect
to notification released by a government
(c) All three concerning the Pollution Index:
(d) None 1. A new categorization of industries based
on their pollution load has been released
2. Consider the following pairs: by the Ministry of Environment, Forest
and Climate Change (MoEFCC).
Technologies Used for
2. The new criteria will prompt industrial
1. Gravitational To control air

RE
sectors willing to adopt cleaner
settling pollution
technologies, ultimately resulting in
2. Phytoremediation To control noise generation of more pollutants but in
pollution limit.
3. Electrostatic To control water 3. Red category of industries are normally
precipitators pollution be permitted in the ecologically fragile
How many of the above pairs is/are correctly area / protected area.
matched?
O
How many of the above statements is/are
(a) Only one correct?

(b) Only two (a) Only one


SC
(c) All three (b) Only two
(d) None (c) All three
(d) None
3. Which of the following statements regarding
the Biological Oxygen Demand (BOD) is
correct? 6. Consider the following:
GS

(a) It is the measure of oxygen equivalent 1. Bioremediation techniques are destruction


of the requirement of oxidation of total techniques to stimulate the growth of
organic matter present in water. microorganisms, using the contaminants
as a food and energy source.
(b) The higher value of BOD indicates the
presence of higher amount of Dissolved 2. Bio augmentation is the process in
CO
Oxygen in water. which microorganisms are imported
to a contaminated site to enhance the
(c) BOD is available to the biodegradable
materials only. degradation process.
S.

(d) All of the above statements are correct. 3. Bioleaching is the extraction of metals
from their ores through the use of living
TE

organisms.
4. Which of the following pairs regarding the
pollutants and their sources are correctly How many of the above statements is/are
NO

matched? correct?
1. Sulphur compounds : Oil Refineries (a) None
DF

2. Nitrogen compounds : Motor vehicle (b) Only one


exhaust
(c) Only two
3. Carbon compounds : Automobile
.P

exhaust (d) All three


W

2 PPP-PTS-4341/092023/15
W
W
https://www.pdfnotes.co/

7. Which of the following is/are incorrect 10. Consider the following statements with respect
regarding Non-ionising radiations? to Black Carbon:
(a) They damage eyes which may be caused 1. Black Carbon (BC) has emerged as
by reflections from coastal sand, snow a major contributor to global climate
(snow blindness) directly looking towards change.
the sun during an eclipse. 2. It is highly effective in absorbing solar
(b) They have low penetrating power and energy and can absorb energy much more
affect the cells and molecules which times than Carbon Dioxide.
absorb them. 3. Lifetime of black carbon in the atmosphere
is only a few weeks.
(c) The actively dividing cells such as
Embryo are more prone to be damaged 4. It was first identified in the Arctic haze
from Non-ionizing radiations than the aerosols and the Arctic snow.
Ionizing radiations.
How many of the above statements is/are
(d) All of the above correct?
(a) Only one
8. Which of the following prerequisites with
(b) Only two

RE
regard to pollutants are necessary for bio
magnification in the food web? (c) Only three
1. Short Lived (d) All four

2. Solubility in fats
11. Which of the following statement regarding
3. Mobility O thermal pollution is incorrect?
4. Solubility in water (a) The change in temperature impacts
organisms by decreasing oxygen supply.
Select the correct answer using the code given
below: (b) Thermal pollution decreases the
metabolic rate of aquatic animals.
SC
(a) 2 and 4 only
(c) It increases plant growth rates resulting
(b) 1 and 3 only in a shorter life span.
(c) 2 and 3 only (d) It may result in a migration of organisms
to another more suitable environment.
(d) 1, 2, 3 and 4

12. Consider the following statements regarding


9. Which of the following statements regarding Corporate Average Fuel Efficiency (CAFE)
GS

the sewage management steps is/are correct? Regulation:


1. Primary treatment is done to remove Statement-I:
solid matters to prevent the grit from
causing damage to the equipment. These aim at lowering fuel consumption of
vehicles by lowering carbon dioxide (CO2)
CO
2. Aeration creates a suitable environment emissions.
for natural bacteria to digest the waste in
Statement-II:
water and form activated sludge.
S.

The norms are applicable for petrol and diesel


3. The tertiary treatment stage has the vehicles exempting LPG and CNG passenger
ability to remove up to 99 percent of the
TE

vehicles.
impurities from the wastewater.
Which one of the following is correct in respect
Select the correct answer using the code given
NO

of the above statements?


below:
(a) Both Statement-I and Statement-II are
(a) 1 and 3 only correct and Statement-II is the correct
DF

explanation for Statement-I


(b) 2 only
(b) Both Statement-I and Statement-II
(c) 1, 2 and 3 are correct and Statement-II is not the
.P

(d) 1 and 2 only correct explanation for Statement-I


W

PPP-PTS-4341/092023/15 3
W
W
https://www.pdfnotes.co/

(c) Statement-I is incorrect but Statement- Which of the statements above is/are correct?
II is correct
(a) 1 only
(d) Statement-I is correct but Statement-II (b) 2 only
is incorrect
(c) Both 1 and 2
13. The prerequisite for the occurrence of harmful (d) Neither 1 nor 2
algal bloom (HAB) are:
16. With reference to solid waste management
1. Sunlight
consider the following statements:
2. Fast-moving water
1. Incineration is the method of waste
3. Nitrogen management that reduces the volume
of waste up to 20 or 30% of the original
4. Phosphorus volume.
Select the correct answer using the code given 2. Pyrolysis is a method of solid waste
below: management whereby solid wastes are
chemically decomposed by heat in the

RE
(a) 1 and 3 only presence of oxygen.
(b) 2 and 4 only 3. Industries, crop farms, institutions, and
hospitals are some of the sources of solid
(c) 1, 3 and 4 only
waste.
(d) 3 and 4 only
How many of the above statements is/are
correct?
14. PRANA Portal recently launched by Union
O (a) All three
Environment ministry is associated with
which of the following? (b) Only one

(a) Tracking the progress of the National (c) Only two


SC
Clean Air Programme (NCAP) in order (d) None
to comply with India’s commitment
to ensure clean air and blue skies to
17. Consider the following statements:
everybody.
1. Temperature inversion, that causes air
(b) Easing flow of investment for development pollutants to dwell near the surface for
of infrastructure in India’s International a longer time leading to serious health
GS

Blue Flag Certified beaches. risks to all life forms, is a typical feature
of the summer season.
(c) Preventing and tracking the man-animal
conflict in elephant corridor regions. 2. PM2.5, due to their small size, reaches
deeper into the respiratory system as
(d) Promoting spread of organic culture in compared to PM10 or larger particles.
CO
agriculture of sea-weed for consumption
purposes. 3. The pollutants emitted by engines such
as Nitrogen oxides (NOx) and Volatile
Organic Compounds (VOC) in the
S.

15. With reference to Hazardous Waste presence of sunlight produce ozone as a


Management, consider the following secondary pollutant.
TE

statements:
How many of the above statements is/are
1. It constitutes any waste which because correct about the various effects of air pollution
NO

of characteristics, such as physical, at the global as well as local level?


chemical, biological, reactive, toxic, (a) All three
flammable, explosive, or corrosive, causes
DF

danger to health, or the environment. (b) Only one

2. India does not have dedicated guidelines (c) Only two


.P

for the management of such waste. (d) None


W

4 PPP-PTS-4341/092023/15
W
W
https://www.pdfnotes.co/

18. Which of the following statements correctly 3. Loss of soil moisture and useful
describe the technologies that have helped microbes.
to reduce air pollution by replacing their
predecessors? 4. Reduced lung function.

1. Use of LED bulbs replacing the How many of the above are the possible
incandescent bulbs that consumed 4-5 impacts of stubble burning?
times more power for the same intensity
of the light. (a) Only one

2. The use of diesel engine tractors for (b) Only two


farming replacing the use of animals, as
(c) Only three
rearing livestock causes enteric methane
emission (EME) which makes up 30% (d) All four
of the total anthropogenic methane
emission.
21. Which of the following is not a technique that
3. Use of LPG for cooking food as compared can be used for the removal of pollutants from
to biogas fuel.
water?
Select the correct answer using the following

RE
(a) Reverse osmosis.
code:
(a) 1 only (b) Semiconductor photo catalysis.

(b) 1 and 2 only (c) Electrolysis.

(c) 3 only (d) Saponification


(d) 1, 2 and 3
O
19. With respect to the concept of Dead zones in
the oceans, consider the following statements:
22. Which of the following statements describe
the impact of environmental degradation on
food production.
SC
1. It is caused due to excessive nutrient 1. An increase in atmospheric CO2 is
pollution from human activities that beneficial for some crops to avoid the
deplete the oxygen required to support losses due to photo respiratory damages
the marine ecosystem. provided the greenhouse effect of CO2
2. These occur near inhabited coastlines, remains within the tolerance level.
where aquatic life is most concentrated.
2. Emissions from fossil fuel burning
3. There are many physical, chemical, and accelerate the formation of ground-level
GS

biological factors that combine to create ozone that diminishes the oxidative
dead zones. properties of the molecular oxygen and
How many of the above statements is/are prevents photo respiratory damage to the
correct? plants.
CO
(a) All three 3. A rise in ambient temperature also
(b) Only one disrupts the germination of many crops
as they require a certain duration of cold
S.

(c) Only two treatment (seed stratification) to break


TE

(d) None the seed dormancy.

Select the correct answer using the following


20. Consider the following:
NO

code:
1. Emission of Volatile Organic Compound
(a) 1 and 2 only
(VOC) and carcinogenic polycyclic
DF

aromatic hydrocarbons into the (b) 2 and 3 only


atmosphere.
(c) 1 and 3 only
2. Increase in soil organic carbon and soil
.P

fertility. (d) 1, 2 and 3


W

PPP-PTS-4341/092023/15 5
W
W
https://www.pdfnotes.co/

23. Consider the following statements: How many of the above factors is/are
responsible for ocean acidification?
1. Degradation of soil due to the storage
or dumping of untreated industrial (a) Only one
wastewater may lead to the pollution of (b) Only two
aquifers permanently.
(c) All three
2. The dust produced from the mining
(d) None
activities may render the top-soil
unusable for some time but they are easy
to remove since they do not enter the 26. Which of the following statements regarding
soil. UN Environment Programme (UNEP) is/are
correct?
3. Accidental oil spills affect soil fertility 1. It established Intergovernmental Panel
severely but due to its low permeability on Climate Change (IPCC) in 1988
in the soil, the deep soil layers remain along with the World Meteorological
unaffected. Organization.
How many of the above statements is/are 2. It hosts the secretariats of Bamako
correct about soil pollution and its ways to Convention and Carpathian Convention.

RE
reach the human system? 3. It publishes the Global Environment
Outlook report.
(a) Only one
Select the correct answer using the code given
(b) Only two below:
(c) All three (a) 1 only
(d) None (b) 2 and 3 only
O
(c) 1 and 2 only
24. Which of the following are the extra-terrestrial
causes of climate change? (d) 1, 2, and 3
SC
1. Fluctuation in the amount of energy 27. Consider the following statements in the
irradiated from the photosphere. context of Arctic Amplification:
2. Increase or decrease the number of 1. It refers to greater climate change near
sunspots. the arctic as compared to the rest of the
hemisphere or globe.
3. Variations in the earth’s orbital
characteristics around the sun. 2. Changes in albedo and ocean currents are
possible reasons for arctic amplification.
GS

4. Changes in the earth’s axial tilt.


3. As compared to Arctic, Antarctic warming
Select the correct answer using the code given has been similar to the global average.
below: How many of the above statements is/are
(a) 1 and 2 only correct?
CO
(a) None
(b) 3 and 4 only
(b) Only one
(c) 1, 2 and 4 only
S.

(c) Only two


(d) 1, 2, 3 and 4
(d) All three
TE

25. Consider the following:


28. The term “internal carbon price” relates to:
NO

1. Shallowing of Carbonate Compensation


(a) A system where emitters can trade
Depth (CCD)
emission units to meet their emission
2. Increase in oceans’ capacity to absorb targets.
DF

carbon dioxide (b) A price on carbon by defining an explicit


tax rate on GHG emissions or more
3. Thickening of shells in marine
.P

commonly on the carbon content of fossil


organisms fuels, i.e. a price per tCO2e.
W

6 PPP-PTS-4341/092023/15
W
W
https://www.pdfnotes.co/

(c) A tool an organization uses internally How many of the above statements is/are
to guide its decision-making process in correct?
relation to climate change impacts, risks (a) Only one
and opportunities.
(b) Only two
(d) A funding approach where payments
(c) All three
are made after pre-defined outputs or
outcomes related to managing climate (d) None
change.
32. Cultivation of plants in agriculture sequesters
large amount of carbon, but at global scale
29. Select the most appropriate definition of agriculture is still source of atmospheric
‘Global Dimming’ a phenomenon often seen in carbon. Which of the following is/are the
news: correct reason (s) for this?
(a) It refers to the collective efforts of G20 1. Animal husbandry is a large source of
countries to reduce the global carbon atmospheric carbon such as methane and
carbon dioxide.
footprint.
2. Ploughing and flood irrigation like

RE
(b) It is an initiative of the UNEP aimed at practices cause release of the carbon
reducing the urban heat island effect by fixed in the soil.
propagating dim colours for high rises in
Select the correct answer using the codes given
urban areas. below:
(c) It is a drop in the amount of average (a) 1 only
solar energy reaching the earth’s surface
O (b) 2 only
due to a shield created by suspended air
particulate materials. (c) Both 1 and 2

(d) An initiative of the UNFCCC, to reduce (d) Neither 1 nor 2


SC
global burning of fossil fuels and shift to
cleaner sources of energy. 33. Which of the following is not a geoengineering
technique to counter climate change by
reducing atmospheric carbon at global scale?
30. Environmental Impact Assessment in India
gets its statutory backing from: (a) Albedo enhancement
(b) Rooftop solar-energy
(a) Environmental (Protection) Act, 1986
(c) Ocean fertilization
GS

(b) Forest Conservation Act, 1980


(d) Enhanced weathering
(c) Wildlife Protection Act, 1972

(d) Air (Prevention and Control of Pollution) 34. After successful global effort to limit the use of
chlorofluorocarbon (CFC), what are the major
Act, 1981
CO
factors responsible for slowing down of the
stratospheric ozone (O3) recovery?
31. Consider the following statement regarding
(a) Emission of Greenhouse gases (GHGs)
SLACC (sustainable livelihood and adaption
S.

like CO2 and CH4 in large amount.


to climate change):
(b) Scarcity of molecular oxygen due to high
TE

1. The SLACC project is supported by the chemical and biological oxygen demand
World Bank. by water bodies.
NO

2. National Institute of Rural Development (c) Increase in infrared waves in solar


incoming radiation than UV rays (needed
and Panchayati Raj is the implementing
for ozone production).
agency of this project.
DF

(d) Excessive telecommunication radio wave


3. Objective of SLACC is to support urban radiation, that caused destruction of
entrepreneurs to develop viable business ozone (O3) into molecular oxygen (O2) and
.P

solutions to mitigate climate change. atomic/free oxygen (O).


W

PPP-PTS-4341/092023/15 7
W
W
https://www.pdfnotes.co/

35. Which of the following statements regarding How many of the above are the benefits of
carbon offsetting initiatives is/are correct? artificial reefs?

1. Air Carbon Exchange (ACX) is the (a) Only one


first exchange in Asia to list Certified (b) Only two
Emission Reductions (CER) held in the
Clean Development Mechanism registry. (c) Only three

2. Gold Standard Voluntary Emission (d) All four


Reductions (VER) is a certificate issued
by the United Nations to member nations 38. Consider the following statements regarding
for preventing one tonne of carbon dioxide UN REDD+ (Reducing emissions from
emissions. deforestation and forest degradation):

Select the correct answer using the code given Statement-I:


below: It is a knowledge and advisory platform on the
forest solutions been built upon the cooperation
(a) 1 only
among and Agriculture Organization of the

RE
(b) 2 only United Nations, United Nations Development
Programme and the United Nations
(c) Both 1 and 2 Environment Programme.
(d) Neither 1 nor 2 Statement-II:
It has set the target of reducing forest emissions
36. Climate change can affect agriculture through
by 10 gigatons per year from 2025.
their direct and indirect effects, what could be
its likely consequence/consequences?
O Which one of the following is correct in respect
of the above statements?
1. The yields of some cereal crops like wheat
are likely to be increased due to increase (a) Both Statement-I and Statement-II are
SC
in crop growth duration. correct and Statement-II is the correct
explanation for Statement-I
2. It can affect the survival and distribution
(b) Both Statement-I and Statement-II
of pest populations and thus developing
are correct and Statement-II is not the
a new equilibrium between crops and
correct explanation for Statement-I
pests.
(c) Statement-I is incorrect but Statement-
3. Increase in atmospheric carbon dioxide II is correct
GS

has a fertilization effect on crops and thus


promotes their growth and productivity. (d) Statement-I is correct but Statement-II
is incorrect
Select the correct answer using the code given
below: 39. Consider the following statements regarding
CO
(a) 1 and 2 only Global Warming Potential (GWP):

(b) 2 and 3 only 1. It is the heat absorbed by any greenhouse


gas in the atmosphere, as a multiple of
S.

(c) 1 and 3 only the heat that would be absorbed by the


TE

same mass of carbon dioxide.


(d) 1, 2 and 3
2. Fluorinated gases have low global
NO

37. Consider the following: warming potentials (GWPs) relative to


other greenhouse gases.
1. Promoting marine biodiversity
3. The calculation of Global Temperature
DF

2. Coastal protection Potential is more complicated than that


for the GWP as it requires modelling
3. Increasing fish catch
of the climate system’s responds to
.P

4. Preventing trawling increased concentrations of GHGs.


W

8 PPP-PTS-4341/092023/15
W
W
https://www.pdfnotes.co/

How many of the above statements is/are 2. National Adaptation Fund (NAF) was
incorrect? later renamed as National Adaptation
Fund for Climate Change (NAFCC) in
(a) Only one
2015, to meet the cost of adaptation to
(b) Only two climate change for the State and Union
Territories.
(c) All three
(d) None Select the correct answer using the following
code:

40. Consider the following statements about (a) 1 only


Ground-level ozone:
(b) 2 only
1. It is an air pollutant and is harmful to
(c) Both 1 and 2
both human as well as vegetation.
(d) Neither 1 nor 2
2. It is not emitted directly into the air, but
is created by chemical in the presence of
sunlight. 43. With reference to Coastal Fisheries Initiative,
consider the following statements:

RE
Which of the above statements is/are correct?
1. It is an initiative of the World Bank that
(a) 1 only
aims to ensure that coastal fisheries
(b) 2 only contribute to food security as well as
economic and social development.
(c) Both 1 and 2
2. It is funded by the Global Environment
(d) Neither 1 nor 2 O Facility (GEF).

41. Consider the following pairs regarding the Which of the above statements is/are
Greenhouse Gas Removal techniques: incorrect?
(a) 1 only
SC
Techniques Procedure
1. Ocean Fertili- Adding nutrients to the (b) 2 only
sation ocean in selected locations to
increase primary production (c) Both 1 and 2
2. Enhanced Exposing large quantities (d) Neither 1 nor 2
Weathering of minerals that will react
with carbon dioxide in the
atmosphere 44. CRISP-M Tool, seen in news recently is a:
GS

3. Ocean Directly ameliorate ocean (a) New genome editing scissor


Alkalinity acidification
(b) Tool for integration of climate information
Enhancement
in Mahatma Gandhi NREGA
How many of the above pairs is/are correctly
CO
matched? (c) New state of the art data platform for
Ocean Data Management
(a) Only one
(d) Tool to facilitate the planning of new
S.

(b) Only two MGNREGA assets using Remote Sensing


and GIS-based information
TE

(c) All three


(d) None
45. Consider the following statements:
NO

42. Which of the following statements is/are 1. A wildfire is an unplanned fire that
correct about climate financing in India? burns in a natural area such as a forest,
DF

grassland, or prairie.
1. National Clean Energy Fund is funded
by international bodies like World Bank 2. The risk of wildfires increases in extremely
Green Energy Fund and Environment dry conditions, such as drought, and
.P

Cess collected from Indian taxpayers. during high winds.


W

PPP-PTS-4341/092023/15 9
W
W
https://www.pdfnotes.co/

Which of the above statements is/are correct? 2. Its mandate is to promote the mandatory
use of carbon market mechanisms.
(a) 1 only
3. The Climate Neutral Now Initiative has
(b) 2 only
no participation fee.
(c) Both 1 and 2
4. A participant can stop participating at
(d) Neither 1 nor 2 any point in time.
How many of the statements above is/are
46. Regarding Forest and Climate Leaders’ incorrect?
Partnership (FCLP), consider the following
statements: (a) Only one

1. FCLP was launched at COP26 in (b) Only two


Glasgow. (c) Only three
2. Its aim to accelerate global progress to (d) All four
halt and reverse forest loss and land
degradation by 2030.
49. Consider the following statements:
3. The FCLP works with existing initiatives

RE
and organisations to deliver ambition in 1. The Accelerating to Zero coalition (A2Z)
3 Action Areas. was launched at COP27.

How many of the above statements is/are 2. It will work towards all sales of new cars
correct? and vans being zero emission globally by
2030 and by no later than 2025 in leading
(a) Only one markets.
(b) Only two
O 3. It builds off the foundation of the ‘Zero
(c) All three Emission Vehicles Declaration’ (ZEV
Declaration) generated at COP21.
(d) None
How many of the above statements is/are
SC
correct?
47. Consider the following statements:
(a) Only one
1. India’s historical contribution to
cumulative global GHG emissions is (b) Only two
therefore minuscule despite having a
share of 17% of the world’s population. (c) All three

2. India’s per capita annual emissions are (d) None


about a third of the global average.
GS

50. Which of the following are the environmental


3. India plans to harness its abundant
effects of stratospheric ozone depletion?
solar, wind, and hydro resources to scale
up renewable energy generation. 1. It causes damage to early developmental
stages of fish, shrimp, crab, amphibians,
How many of the statements given above is/
CO
and other marine animals.
are correct?
2. Alters both sources and sinks of
(a) Only one
greenhouse and chemically important
S.

(b) Only two trace gases.


(c) Only three
TE

3. UVB radiation affects the physiological


and developmental processes of plants.
(d) All four
How many of the above is/are the environmental
NO

48. With reference to Climate Neutral Now effects of stratospheric ozone depletion?
Initiative, consider the following statements: (a) Only one
DF

1. It was launched by the United Nations (b) Only two


Environment Programme (UNEP) to
increase climate action by engaging non- (c) All three
.P

Party stakeholders.
(d) None
W

10 PPP-PTS-4341/092023/15
W
W
https://www.pdfnotes.co/

Space for Rough Work

RE
O
SC
GS

CO
S.
TE
NO
DF
.P
W

PPP-PTS-4341/092023/15 11
W
W
https://www.pdfnotes.co/

IAS Prelims - 2024


PPP-PTS-4341/092023/15

GENERAL STUDIES

ENVIRONMENT - 4

Answer Key

Q. 1 (c) Q. 11 (b) Q. 21 (d) Q. 31 (b) Q. 41 (c)

Q. 2 (a) Q. 12 (d) Q. 22 (c) Q. 32 (c) Q. 42 (d)

Q. 3 (c) Q. 13 (c) Q. 23 (a) Q. 33 (b) Q. 43 (d)

Q. 4 (d) Q. 14 (a) Q. 24 (d) Q. 34 (a) Q. 44 (b)


CO
Q. 5 (a) Q. 15 (a) Q. 25 (b) Q. 35 (a) Q. 45 (c)

Q. 6 (d) Q. 16 (c) Q. 26 (d) Q. 36 (b) Q. 46 (a)


S.

Q. 7 (c) Q. 17 (b) Q. 27 (d) Q. 37 (d) Q. 47 (c)


TE

Q. 8 (c) Q. 18 (a) Q. 28 (c) Q. 38 (d) Q. 48 (b)


NO

Q. 9 (c) Q. 19 (a) Q. 29 (c) Q. 39 (a) Q. 49 (a)

Q. 10 (d) Q. 20 (b) Q. 30 (a) Q. 40 (c) Q. 50 (c)


DF
.P
W
W
W
https://www.pdfnotes.co/

1. Correct Option: (c)  Biological Oxygen Demand is the amount


of dissolved oxygen needed by bacteria in
Explanation: decomposing the organic wastes present in
water.
 Option (c) is correct
 It is expressed in milligrams of oxygen
Supplementary Notes per litre of water.
Fly Ash  The higher value of BOD indicates low
Dissolved Oxygen content of water.
 Fly ash is the finely divided residue that
results from the combustion of pulverized  BOD is limited to biodegradable materials
only; hence it is not considered a much
coal and is transported from the combustion
reliable method of measuring pollution
chamber by exhaust gases. load in the water.
 It is produced from the combustion of coal  Chemical oxygen demand (COD) is a slightly
in electric utility or industrial boilers, better mode used to measure pollution load
produced by coal-fired electric and steam in the water. COD is the measure of
generating plants. oxygen equivalent to the requirement
of oxidation of total organic matter (i.e.
 It consists primarily of oxides of silicon,
biodegradable and non-biodegradable)
aluminium, iron, and calcium. present in water.
 Magnesium, potassium, sodium, titanium, L2: Approachable
and sulphur are also present to a lesser

RE
degree.
4. Correct Option: (d)
L2: Approachable
Explanation:
2. Correct Option: (a)  All pairs are correctly matched
Supplementary Notes
Explanation:
O
 Pair 1 is correctly matched: Gravitational Pollutants and their sources
settling is used to control air pollution. It is  Nitrogen Compound
used to remove large diameter suspended Source: Motor vehicle exhaust
particles from a gas by using gravitational
 Effects:
SC
force.
 Irritation in eyes and lungs (NO and
 Pair 2 is incorrectly matched: N2O) atmospheric reaction
Phytoremediation is used to control soil
 Low productivity in plants
pollution. It uses microorganisms (bacteria
and fungi) to damage the environment into  Acid rain damages material (metals and
less toxic form. The microorganisms may be stone)
native to the polluted area or they may be  Sulphur compounds
invasive. Source: Power plants and refineries
GS

 Pair 3 is incorrectly matched:  Effects:


Electrostatic precipitators are the control
 Respiratory problems in humans (SO2
measure of air pollution. Works on the and H2S) volcanic eruptions
principle of electrostatic precipitation, in
Loss of chlorophyll in plants (chlorosis)
CO
which electrically charged particulates 
existing in the polluted gas are isolated  Acid rain
from the gas stream under the influence of  Hydrocarbons (benzene, ethylene)
S.

electrical field.
Source: Automobiles and petroleum
L3: Real Challenge industries
TE

 Effects:
3. Correct Option: (c)  Respiratory problem
NO

 Cancer causing properties


Explanation:
 Carbon compound (CO and CO2)
 Option (c) is correct
Source: Automobile exhaust, burning of
DF

Supplementary Notes wood and coal


Biological Oxygen Demand (BOD)  Effects:
.P

 Water pollution by organic wastes is  Respiratory problems


measured in terms of Biochemical Oxygen  Greenhouse effect
W

Demand (BOD). L3: Real Challenge


W

2 PPP-PTS-4341/092023/15
W
https://www.pdfnotes.co/

5. Correct Option: (a) 6. Correct Option: (d)


Explanation: Explanation:
 Statement 2 is incorrect: The new criteria  All statements are correct
will prompt industrial sectors willing to
adopt cleaner technologies, ultimately Supplementary Notes
resulting in generation of fewer pollutants. Bioremediation
 Statement 3 is incorrect: Red category of  Bioremediation techniques are destruction
industries normally are not permitted in the techniques to stimulate the growth of
ecologically fragile area / protected area. microorganisms, using the contaminants as
Supplementary Notes a food and energy source.
 These techniques have been successfully used
Pollution Index
to remediate soils/sludge & groundwater
 The Ministry of Environment, Forest and contaminated by petroleum hydrocarbons,
Climate Change (MoEFCC) has developed solvents, pesticides, wood preservatives,
the criteria of categorization of industrial and other organic chemicals.
sectors based on the Pollution Index which is
a function of the emissions (air pollutants),  Oxygen, water & nutrients are added, and
effluents (water pollutants), hazardous the temperature and pH are controlled.
wastes generated and consumption of The rate at which microorganisms degrade

RE

resources. The Pollution Index PI of any the contaminants is influenced by: the specific
industrial sector is a number from 0 to 100 contaminants present, their concentrations,
and the increasing value of PI denotes the the oxygen supply, moisture, temperature,
increasing degree of pollution load from the pH, nutrient supply, bio-augmentation, and
industrial sector. co-metabolism.
 The new criteria will prompt industrial  Micro-organisms can be adapted to degrade
sectors willing to adopt cleaner technologies, specific contaminants or enhance the
O
ultimately resulting in generation of less process.
pollutants.
In situ bioremediation
 ‘Range of Pollution Index ‘for the purpose of
categorization of industrial sectors is:  Bioventing: supply of air and nutrients
SC
through wells to contaminated soil to
 Industrial Sectors having Pollution Index
stimulate the growth of indigenous bacteria.
score of 60 and above - Red category
It is used for simple hydrocarbons and can
 Industrial Sectors having Pollution Index be used where the contamination is deep
score of 41 to 59 - Orange category under the surface.
 Industrial Sectors having Pollution  Biosparging: Injection of air under
Index score of 21 to 40 - Green category pressure below the water table to increase
 Industrial Sectors having Pollution Index groundwater oxygen concentrations and
GS

score incl. & up to 20 - White category enhance the rate of biological degradation
of contaminants by naturally occurring
 No Red category of industries shall normally
bacteria.
be permitted in the ecologically fragile area
/ protected area.  Bio augmentation: Microorganisms are
CO
imported to a contaminated site to enhance
 There shall be no necessity of obtaining
the degradation process.
the Consent to Operate for White category
of industries. An intimation to concerned  Bioleaching: It is the extraction of metals
S.

SPCB / PCC shall suffice from their ores through the use of living
organisms. This is much cleaner than the
TE

Some examples: traditional heap leaching using cyanide.


 White: Air conditioners, bicycles, CFL
Ex-situ bioremediation
NO

lamps, Organic manure, etc.


 Green: Aluminium Utensils, Distilled  Bioreactors: These are large vessels where
water, flour mills, Ayurvedic medicines, the contaminated material can be monitored
and conditions for bioremediation can be
DF

 Orange: Dry cell battery, food processing, controlled. Biological organisms typically
glass manufacturing, and automobile have conditions where they operate best.
servicing In bioreactors we can control the mixing
.P

 Red: Lead acid battery, cement, oil & gas rate, temperature, pH, and nutrient levels
extraction, pharmaceuticals, to suit the organisms breaking down our
W

L3: Real Challenge contaminants.


W

PPP-PTS-4341/092023/15 3
W
https://www.pdfnotes.co/

 Land farming: It involves spreading  The mutated gene can persist in living
contaminated soil into a lined bed (to organisms and may affect their progeny.
prevent leaching) and periodically applying  The actively dividing cells such as Embryo,
nutrients and mixing the soil to boost foetus, cells of the skin, intestinal lining,
biological activity. bone marrow, and gamete forming cells are
 Bio piling: It places the contaminated soil more sensitive to radiations.
into piles that are well aerated and nutrients
 Some species of animals and plants
are added to speed up bioremediation. In all
preferentially accumulate specific
cases, the contaminant levels are monitored
radioactive materials. For example, oysters
to verify that bioremediation is taking
deposit 65Zn, fish accumulate 55Fe, and
place and steps are taken to ensure that
marine animals selectively deposit 90Sr.
contaminated material stays out of contact
with the environment. L2: Approachable
L3: Real Challenge
8. Correct Option: (c)
7. Correct Option: (c) Explanation:
Explanation:  Option (c) is correct
 Option (c) is correct Supplementary Notes

RE
Supplementary Notes Bio magnification
Two types of radiations with regard to  Bio magnification refers to the tendency
the mode of their action on cells of pollutants to concentrate as they move
from one trophic level to another. Thus it is
 Radiation in the ultraviolet band and at
an increase in concentration of a pollutant
lower energies (to the left of ultraviolet) is
from one link in a food chain to another.
called non-ionizing radiation, while at the
higher energies to the right of the ultraviolet  In order for bio magnification to occur, the
O
band is called ionizing radiation. pollutant must be:
Non-ionising radiations  Long lived: If a pollutant is short-lived,
it will be broken down before it can
 Examples include Radiofrequency (RF) become toxic. Long life of a pollutant
radiation, Microwaves, Infrared radiation, allows it to sustain itself as it moves up
SC
and Ultraviolet (UV). the food chain.
 They have low penetrating power and  Mobile: Immobility of a pollutant
affect the cells and molecules which restricts it to one position and thus
absorb them. becomes unlikely to be taken up by
 They damage eyes which may be caused the organism thus hindering the bio
by reflections from coastal sand, snow magnification.
(snow blindness) directly looking  Soluble in fats: Pollutants that dissolve
towards the sun during an eclipse.
GS

in fats are retained for a long time as the


 They injure the cells of skin and blood breakdown of such pollutants becomes a
capillaries producing blisters and bit complex.
reddening called sunburns.  Biologically active: If a pollutant is not
biologically active, it may bio magnify but
CO
Ionizing radiations
is not toxic to a great degree. Example:
 They include X-rays, cosmic rays and DDT
atomic radiations (radiations emitted by
Less soluble in water: If the pollutant
S.

radioactive elements). 
is soluble in water, it has the possibility
 Ionizing radiations have high
to be excreted by the organism.
TE

penetration power and cause breakage of


macromolecules.  Example: Veterinary use of the non-
steroidal anti-inflammatory (NSAID)
The molecular damage may produce short-
NO


drug diclofenac in South Asia has resulted
range (immediate) or long-range (delayed)
in the collapse of populations of vulture
effects.
species of the genus Gyps to the most
 Short-range effects include burns, impaired severe category of global extinction risk.
DF

metabolism, dead tissues and death of the Vultures are exposed to diclofenac when
organisms. scavenging on livestock treated with the
Long-range effects are mutations that drug shortly before death. Diclofenac causes
.P


increased the incidence of tumours kidney damage, increased serum uric acid
and cancer, shortening of life-span and concentrations, visceral gout, and death.
W

developmental changes. L2: Approachable


W

4 PPP-PTS-4341/092023/15
W
https://www.pdfnotes.co/

9. Correct Option: (c)  Reducing CO2 emissions is essential to avert


the worst impacts of future climate change,
Explanation: but CO2 has such a long atmospheric
 All statements are correct lifetime that it will take several decades for
CO2 concentrations to begin to stabilize after
Supplementary Notes
emissions reductions begin. In contrast, BC
Sewage Treatment Stages remains in the atmosphere for only a
 Primary treatment is done by pouring few weeks, so cutting its emissions
the wastewater into big tanks for the would immediately reduce the rate of
solid matter to settle at the surface warming, particularly in the rapidly
of the tanks. The sludge, the solid waste changing Arctic. Moreover, reduced
that settles at the surface of the tanks, is exposure to BC provides public health co-
removed by large scrapers and is pushed benefits, especially in developing countries.
to the center of the cylindrical tanks and Technologies that can reduce global BC
later pumped out of the tanks for further emissions are available today.
treatment. It is done to remove metals to
prevent the grit from causing damage  Being the strongest absorbent of light
to the equipment. The remaining water is amongst all the components of particulate
then pumped for secondary treatment. matter. It has the ability to reduce albedo
when deposited on snow. It thus warms the
 The secondary treatment stage involves
region by reflecting less solar energy.
adding seed sludge to the wastewater to

RE
ensure that it is broken down further. Main Properties of Black Carbon
Air is first pumped into huge aeration tanks
which mix the wastewater with the seed  It is directly emitted in the atmosphere as
sludge which is basically a small amount of fine particles (PM5.)
sludge, which fuels the growth of bacteria  It is highly effective in absorbing solar
that uses oxygen and the growth of other energy and can absorb million times more
small microorganisms that consume the
energy than Carbon Dioxide.
remaining organic matter. This process
O
leads to the production of large particles  It is formed of pure carbon in several linked
that settle down at the bottom of the huge forms.
tanks.
 It is emitted in two forms-anthropogenic
 The tertiary treatment stage has the and soot. It is a major component of soot.
SC
ability to remove up to 99 percent of
the impurities from the wastewater.  Climatologically, it is highly active and is
This produces effluent water that is close known as a climate forcing agent. It adds
to drinking water quality. Unfortunately, to global atmospheric heating by reducing
this process tends to be a bit expensive as the albedo which is the property of snow
it requires special equipment, well trained and ice to reflect sunlight.
and highly skilled equipment operators,
 BC emissions are largely done from
chemicals and a steady energy supply. All
automobile sources like diesel engines
these are not readily available.
GS

and other vehicles. This comprises 52% of


L3: Real Challenge total BC emissions. Other sources include
burning of biomass, residential processes,
10. Correct Option: (d) industrial heating etc.
CO
 Largest contributors of BC in the world are
Explanation:
Asia, some parts of the African continent and
 All statements are correct Latin America. China and India together
contribute 25-30% of the total global black
S.

Supplementary Notes
carbon emissions. These emissions are more
Black Carbon intense in the source regions which include:
TE

 Black Carbon (BC) has recently emerged Indo-Gangetic plains of India, Eastern
as a major contributor to global climate China, Southeast Asia and Indonesia,
NO

change, possibly second only to CO2 as the Equatorial Africa, Latin America etc. A
main driver of change. BC particles strongly large part of people live in these hotspots.
absorb sunlight and give soot its black
color.  It was first identified in the Arctic haze
DF

aerosols and the Arctic snow.


 BC is produced both naturally and
by human activities as a result of the  It comprises about 30% of the total
incomplete combustion of fossil fuels, carbon component of soils. It serves a
.P

biofuels, and biomass. Primary sources major reservoir of nutrients for tropical
include emissions from diesel engines, cook soils.
W

stoves, wood burning and forest fires. L3: Real Challenge


W

PPP-PTS-4341/092023/15 5
W
https://www.pdfnotes.co/

11. Correct Option: (b) Supplementary Notes

Explanation: Corporate Average Fuel Efficiency


(CAFE) Regulation
 Option (b) is correct
 Carmakers from the Society of India
Supplementary Notes Automobile Manufacturers (SIAM) recently
requested Ministry for Road Transport to
Thermal pollution
postpone implementation of BS VI CAFE
 Thermal pollution is the rise or fall in Phase II regulations since the industry is
the temperature of a natural aquatic still recovering from the impact of COVID.
environment caused by human influence.  CAFE norms aim at lowering fuel
 Thermal pollution is caused by either consumption of vehicles.
dumping hot water from factories and power  It is achieved by lowering carbon
plants or removing trees and vegetation that dioxide (CO2) emissions.
shade streams, permitting sunlight to raise
the temperature of these waters, release of  Corporate Average refers to sales-
volume weighted average for every auto
cold water which lowers the temperature.
manufacturer.
 The change in temperature impacts
 The norms are applicable for petrol,
organisms by decreasing oxygen
diesel, and LPG and CNG passenger
supply. Warm water contains less oxygen.

RE
vehicles.
Elevated temperature typically decreases
the level of dissolved oxygen (DO) in water.  Upgrading to stricter fuel standards is one
way to tackle air pollution.
 So there is decrease in rate of decomposition
of organic matter. Green algae are replaced  CAFE norms assume importance in the
by less desirable blue green algae. light of their ability to reduce the carbon
footprint.
 It also increases the metabolic rate of
Stricter CAFE targets can also lead to
O
aquatic animals, results in consumption 
of more food in a shorter time than if their manufacturers moving to electric or strong
hybrid.
environment were not changed.
 Corporate Average Fuel Economy (CAFE)
 An increased metabolic rate may result
norms are embedded in BS-VI emission
SC
in food source shortages, causing a sharp
regulations and are part of the government’s
decrease in a population.
efforts to reduce vehicular carbon footprint.
 Primary producers are affected by warm
 CAFE norms require average corporate
water because higher water temperature CO2 emissions to be less than 130 gm/km.
increases plant growth rates, resulting In Phase II (2022 onwards), emissions need
in a shorter life span and species to be further reduced to less than 113 gm/
overpopulation. km.
This can cause an algae bloom which
GS

 L2: Approachable
reduces the oxygen levels in the water. The
higher plant density results in reduced light
intensity, decreases photosynthesis and 13. Correct Option: (c)
leads to an increased plant respiration rate.
Explanation:
CO
This is similar to the eutrophication.
 Option (c) is correct
 It also effects ecosystem composition as
changes in the environment may also result Supplementary Notes
S.

in a migration of organisms to another


Harmful algal blooms
more suitable environment which leads
TE

to competition for fewer resources thereby  Nutrient pollution from human activities
compromising food chains of the old and makes the problem worse, leading to more
NO

new environments. severe blooms that occur more often.


L1 : Elementary  Under the right conditions, algae may grow
out of control — and a few of these “blooms”
produce toxins that can kill fish, mammals
DF

12. Correct Option: (d) and birds, and may cause human illness or
even death in extreme cases. Other algae
Explanation: are nontoxic, but eat up all of the oxygen in
.P

 Statement 2 is incorrect: The norms are the water as they decay, clog the gills of fish
applicable for petrol, diesel, and LPG and and invertebrates, or smother corals and
W

CNG passenger vehicles. submerged aquatic vegetation. Still others


W

6 PPP-PTS-4341/092023/15
W
https://www.pdfnotes.co/

discolour water, form huge, smelly piles on Supplementary Notes


beaches or contaminate drinking water.
Collectively, these events are called harmful Hazardous Waste Management
algal blooms, or HABs.  The management of hazardous waste
is a process that includes the collection,
Harmful algal blooms need: recycling, treatment, transportation,
 Sunlight disposal, and monitoring of wastes disposal
 Slow-moving water sites. In the current scenario of developing
countries, hazardous wastes are often
 Nutrients (nitrogen and phosphorus) disposed of directly into the environment
L2: Approachable posing health and environmental risk.
 Hazardous waste means any waste, which
because of characteristics, such as physical,
14. Correct Option: (a) chemical, biological, reactive, toxic,
Explanation: flammable, explosive, or corrosive, causes
danger to health, or the environment.
 Option (a) is correct
L2: Approachable
Supplementary Notes
PRANA Portal 16. Correct Option: (c)
 Recently Union environment minister

RE
launched a portal called PRANA. Explanation:
 PRANA Portal will be used to track the  Statement 2 is incorrect: Pyrolysis is a
progress of the National Clean Air method of solid waste management whereby
Programme (NCAP) in order to comply solid wastes are chemically decomposed by
with India’s commitment to ensure heat without the presence of oxygen.
clean air and blue skies to everybody.
Supplementary Notes
 This portal was launched on the occasion
O
of ‘International Day of Clean Air for Blue
Skies’ in non-attainment cities (NAC).
Solid-waste management
 Solid-waste management, the collecting,
 Non-attainment cities are those cities that treating and disposing of solid material that
failed to meet the National air quality is discarded because it has served its purpose
standards in a five-year period. or is no longer useful. Improper disposal of
SC
municipal solid waste can create unsanitary
 With India’s efforts, 86 cities showed a conditions, and these conditions, in turn,
better air quality in 2019 and it increased
can lead to pollution of the environment and
to 104 cities in 2020.
outbreaks of vector-borne disease—that is,
 Environment ministry and Central Pollution diseases spread by rodents and insects.
Control Board (CPCB) seeks to achieve 20-
 Incineration is the method that involves the
30 percent reduction in particulate matter
burning of solid wastes at high temperatures
(PM10 as well as PM2.5) concentrations in
until the wastes are turned into ashes.
India by 2024.
GS

Incinerators are made in such a way that


 In order to achieve the target, India has they do not give off extreme amounts of heat
prepared and is implementing city-specific when burning solid wastes.
action plans to improve air quality in 132
NACs/Million Plus Cities.  Pyrolysis is a method of solid waste
CO
management whereby solid wastes are
 It targets city-specific air polluting sources chemically decomposed by heat without
like soil & road dust, vehicles, MSW burning, the presence of oxygen. It usually occurs
domestic fuel, construction material and under pressure and at temperatures of up
S.

industries. to 430 degrees Celsius. The solid wastes are


 PRANA portal will help in tracking physical changed into gasses, solid residue of carbon
TE

and financial status of city air action plan and ash, and small quantities of liquid.
implementation. It will also disseminate L3: Real Challenge
information on air quality to the public.
NO

L2: Approachable
17. Correct Option: (b)
15. Correct Option: (a) Explanation:
DF

 Statement 1 is incorrect: Temperature


Explanation: inversion is the phenomenon of inversion of
.P

 Statement 2 is incorrect: Environment the atmospheric temperature gradient that


Ministry has already Formulated and occurs during the winter season that causes
Notifies Hazardous Waste Management the formation of smog and lets it stay longer
W

Rules, 2016 near the surface.


W

PPP-PTS-4341/092023/15 7
W
https://www.pdfnotes.co/

Supplementary Notes gases (secondary particles) such as sulfur


dioxide (SO2), nitrogen oxides (NOX), and
Temperature Inversion certain organic compounds. These organic
 Usually, the higher the altitude, the lower compounds can be emitted by both natural
the temperature of the air. Temperature sources, such as trees and vegetation, as
inversion is a meteorological phenomenon well as from man-made (anthropogenic)
that develops when cool air is trapped in the sources, such as industrial processes and
ground under a layer of warm air. motor vehicle exhaust.
 There are four types of temperature  Both PM2.5 and PM10 can be inhaled, with
inversions: some depositing throughout the airways,
though the locations of particle deposition
 A ground inversion occurs most often
in the lung depend on particle size. PM2.5
on clear nights when the air close to the is more likely to travel into and deposit on
ground is cooled by radiation. the surface of the deeper parts of the lung,
 A turbulence inversion is created when a while PM10 is more likely to deposit on the
layer of calm air lies above turbulent air. surfaces of the larger airways of the upper
As the turbulent layer brings warm air region of the lung. Particles deposited on
down and cools its upper part through the lung surface can induce tissue damage
what’s called vertical mixing, the layer of and lung inflammation.
calm air above eventually ends up being Formation of Ground Ozone
warmer, thus causing an inversion.

RE
 Tropospheric, or ground-level ozone, is not
 A subsidence inversion forms when emitted directly into the air but is created
a large layer of air sinks due to high by chemical reactions between oxides
pressure and heats up as a result. of nitrogen (NOx) and volatile organic
 A frontal inversion develops when a cold compounds (VOC). This happens when
air mass undercuts a warm air mass and pollutants emitted by cars, power plants,
keeps it on top. industrial boilers, refineries, chemical
plants, and other sources chemically react
O
Temperature Inversions and Smog in the presence of sunlight.
 The warmer air in a temperature inversion  Ozone is most likely to reach unhealthy levels
acts as a lid, trapping pollutants near the on hot sunny days in urban environments,
ground and making them unable to disperse but can still reach high levels during colder
SC
until the weather changes. As a result, the months. Ozone can also be transported long
compounds end up reacting with each other distances by wind, so even rural areas can
to form other pollutants such as ground- experience high ozone levels
level ozone, thus posing a serious risk to L3: Real Challenge
human health.
 Temperature inversions are more common 18. Correct option: (a)
during the winter in the northern
hemisphere due to high pressure, clear Explanation:
GS

skies, and long nights, which allow heat to  Statement 2 is incorrect: Although
dissipate easily from the ground. This leads livestock produce EME, the contribution of
to a phenomenon called winter smog, which diesel engines to air pollution is far more
is further exacerbated by homes burning than livestock. The transition from Bull
CO
wood and coal for warmth. driven farming to engine-driven farming
was mostly due to the efficiency of the
PM10 vs PM2.5 engines.
PM10 and PM2.5 often derive from different
S.

  Statement 3 is incorrect: LPG offers a


emissions sources, and also have different cleaner solution for the household cooking
chemical compositions. Emissions from the
TE

fuel as compared to biomass burning and


combustion of gasoline, oil, diesel fuel or helps to reduce indoor pollution. But when
wood produce much of the PM2.5 pollution it is compared to biogas, which is equally
NO

found in outdoor air, as well as a significant clean and renewable, while the use of LPG
proportion of PM10. PM10 also includes adds net carbon into the atmosphere, LPG
dust from construction sites, landfills and falls short in this context.
agriculture, wildfires and brush/waste
DF

burning, industrial sources, wind-blown Supplementary Notes


dust from open lands, pollen, and fragments LED bulbs vs Incandescent Bulbs
of bacteria.
.P

 LEDs use much less energy than


 PM may be either directly emitted from incandescent bulbs because diode light
sources (primary particles) or formed in the is much more efficient, power-wise, than
W

atmosphere through chemical reactions of filament light.


W

8 PPP-PTS-4341/092023/15
W
https://www.pdfnotes.co/

 LED bulbs use more than 75% less energy 19. Correct Option: (a)
than incandescent lighting. At low power
levels, the difference is even larger. Bright Explanation:
LED flood lamps to use only 11 to 12 watts  All statements are correct
while creating a light output comparable to
a 50-watt incandescent bulb. Supplementary Notes
Benefits of using animals for Farming (although Dead Zones
efficiency is an extremely limiting factor)  Dead zones are hypoxic (low-oxygen)
 Animals are considered to be the backbone areas in the world’s oceans and large
of the rural economy in India. Draught lakes, caused by “excessive nutrient
animals play a dominant role in our rural pollution from human activities coupled
economy. Apart from the manual labour, with other factors that deplete the oxygen
the traditional cultivation in India was required to support most marine life in
based on the use of animal power for 97.6 % bottom and near-bottom water. In the 1970s
of farmers (landowners) accounting for 77.2 oceanographers began noting increased
percent of landholdings. instances of dead zones. There are many
 India had the largest population of draught physical, chemical, and biological
animals in the world (Approx. 88Million). factors that combine to create dead
This localized resource is an exhaustible zones, but nutrient pollution is the
primary cause of those zones created

RE
and available resource with farmers, yet to
be optimized properly. by humans. Excess nutrients that run off
land or are piped as wastewater into rivers
 Animal data (In terms of present
and coasts can stimulate an overgrowth of
Contribution to National Economy and
algae, which then sinks and decomposes
Climate):
in the water. The decomposition process
 Contributes approx. 50 % of all the consumes oxygen and depletes the supply
power consumed in the farming sector in available to healthy marine life.
O
India so approx. annual saving of diesel
is 23.75 MT equivalent to INR 21500
 Less oxygen dissolved in the water is often
referred to as a “dead zone” because most
crore.
marine life either dies, or, if they are mobile
 Gives traction power to 50 million such as fish, leave the area. Habitats that
SC
ploughs in villages. would normally be teeming with life become,
 Gives employment to 20 million people essentially, biological deserts.
on a full-time/part-time basis in the  These occur near inhabited coastlines,
Bullock cart business. where aquatic life is most concentrated due
 Transports approx.15% of the total of the to availability of food and nutritional
load (Tonnes-km) carried by the motor requirements. Dead zones occur in many
transport sector in India. areas of the country, particularly along the
In load terms, bullock carts carry more East Coast, the Gulf of Mexico, and the
GS


load than the total load transported by Great Lakes, but there is no part of the
railways. country or the world that is immune. The
second largest dead zone in the world is
 Provides approx. 100 million tons of dry located in the U.S., in the northern Gulf of
dung per year which is equivalent to INR
CO
Mexico.
5000 crore /year.
L3: Real Challenge
 Saves 5 million tons of firewood per year
which is equivalent to INR 500 crore per
S.

year. 20. Correct Option: (b)


TE

 Provides by-products like skin, bone,  Option (b) is correct


horn, and etc. worth INR 100 crore/
year. Supplementary notes:
NO

 The present market value of draught Stubble Burning


animals @ average 10,000 per pair is  Recently a study conducted in Punjab
INR 30000 crore. by researchers at The Energy Resources
DF

 Gives dung for Bio-gas, cakes & Bio- Institute (TERI), All India Institute of
fertilizers. Medical Sciences (AIIMS), and the Punjab
Agricultural University showed that
.P

 The backbone of rural economy & an


asset in bio-diversity terms. pollution from stubble burning significantly
reduced lung function and was particularly
W

L3: Real Challenge harmful to women in rural Punjab.


W

PPP-PTS-4341/092023/15 9
W
https://www.pdfnotes.co/

 Stubble burning is the act of setting fire to separate ions, unwanted molecules, and
crop residue to remove them from the field larger particles from drinking water.
to sow the next crop. In reverse osmosis, an applied pressure
 To plant the next winter crop (Rabi crop), is used to overcome osmotic pressure,
farmers in Haryana and Punjab have to a colligative property, driven by chemical
move in a very short interval and if they are potential differences of the solvent,
late, due to short winters these days, they a thermodynamic parameter.
might face considerable losses. Therefore,  Semiconductor photo catalysis: This
burning is the cheapest and fastest way to technology is based on the presence of
get rid of stubble. a semiconductor that can be excited by
light with an energy higher than its band
Effects of Stubble Burning gap, inducing the formation of energy-rich
 Pollution: Emits large amounts of toxic electron-hole pairs that can be involved in
pollutants in the atmosphere which contain redox reactions. Photo catalysis offers an
harmful gases like methane (CH4), Carbon interesting approach to water purification,
Monoxide (CO), Volatile Organic Compound offering the possibility of using sunlight
(VOC), and carcinogenic polycyclic aromatic as a sustainable and renewable source of
hydrocarbons. These pollutants disperse energy.
in the surroundings, may undergo a
 Electrolytic wastewater treatment:
physical and chemical transformation, and
Electrolytic wastewater treatment is rarely

RE
eventually adversely affect human health
used in comparison to chemical treatment.
by causing a thick blanket of smog.
However, this treatment is convenient
 Soil Fertility: Burning husk on the ground and may be more efficient to produce high-
destroys the nutrients in the soil, making it quality water. Electrodes with Aluminium
less fertile. (Al), Iron (Fe), Steel (St), and graphite are
 Heat Penetration: Heat generated by generally the best suited to electrochemical
stubble burning penetrates the soil, leading water treatment. In the present review, the
O
to the loss of moisture and useful microbes. applications of electrochemical treatment
as well as electrocoagulation (EC), electro-
 Impact on Respiratory System:
flotation (EF), and electro-coagulation/
Concentrations of PM2.5, the category of
flotation (ECF) to the treatment of
unburnt carbon particles considered most
wastewater and their operating parameters
SC
harmful to respiratory health increase
(reactor design, current density, time, and
during the crop residue burning period
electrode type, and arrangement) affecting
resulting in a two to three-fold increase in
these processes have been discussed. Among
most of the respiratory symptoms including
the electrochemical processes, the EC
wheezing, breathlessness on exertion, cough
process should be the best choice, not only
in the morning, cough at night, skin rashes,
because it can achieve more satisfactory
runny nose or itchiness of eyes, etc. across
removal but also because the process is cost-
all age groups (10-60 years).
effective and simple in the technological
GS

L3: Real Challenge aspect.


 Activated carbon adsorption: Adsorption
21. Correct option: (d) is one of the most effective processes
of advanced wastewater treatment
Explanation:
CO
technologies, that industry and academic
 Option (d) is correct: All the other researchers widely employed for the
techniques except saponification can be used removal of various pollutants. Activated
S.

for the decontamination of polluted water. carbon is one of the most widely investigated
Saponification is the formation of a metallic adsorbents in the water treatment process.
TE

salt of a fatty acid; such a salt is called In recent years, the “adsorption” process
soap. The reaction involves the treatment has become more popular as “biosorption”
of free fatty acids and/or glycerides with a which uses biomaterials as the adsorbent,
NO

base and may be considered a special case of for contaminated water treatment.
hydrolysis when a glyceride is reacted with  Saponification: Saponification is the
a base. formation of a metallic salt of a fatty acid;
DF

Supplementary Notes such a salt is called soap. The reaction


involves the treatment of free fatty acids
Water Treatment Technologies and/or glycerides with a base and may be
.P

 Reverse Osmosis: Reverse osmosis (RO) considered a special case of hydrolysis when
is a water purification process that uses a glyceride is reacted with a base.
W

a partially permeable membrane to L1 : Elementary


W

10 PPP-PTS-4341/092023/15
W
https://www.pdfnotes.co/

22. Correct Option: (c) and experimental evidence demonstrating a


variety of effects at ambient concentrations
Explanation: on crops, forests, and grassland species and
 Statement 2 is incorrect: There are no ecosystems.
beneficial effects of ground ozone formation  Global risk assessment modelling estimates
that have been observed on the crops in yield losses of staple crops between 3 to 16%
contrary to that, ozone causes a wide variety causing economic losses of between US$14
of damage in crops including visible injury, to 26 billion in the year 2000.
reduction in photosynthesis, alterations to
carbon allocation, and reduction in yield  Changes in anthropogenic emissions of
quantity and quality. O3 precursors in recent decades have
modified O3 concentration profiles
Supplementary Notes (peaks versus background O3) and global
distributions with the Northern Hemisphere
CO2 Fertilization Effect seeing increases in O3 levels of between 1
 The CO2 fertilization effect or carbon and 5 ppb/decade since the 1950s and the
fertilization effect causes an increased emergence of Asia as the region with the
rate of photosynthesis while limiting leaf highest O3 concentrations.
transpiration in plants. Both processes result
 In the future, O3 mitigation could focus on
from increased levels of atmospheric carbón
methane (CH4) and nitrogen oxide (NOx)
dioxide (CO2).
emissions; these will differentially influence

RE
 Earth System Models (ESMs), Land System global and local/regional O3 concentrations
Models (LSMs), and Dynamic Global and influence daily and seasonal profiles.
Vegetation Models (DGVMs) are used to
 The consequent effects on vegetation
investigate and interpret vegetation trends
will in part depend on how these changes
related to increasing levels of atmospheric
in O3 profile alter the exceedance of
CO2.
detoxification thresholds for plant damage.
 The carbon fertilization effect varies
O
depending on plant species, air and soil
L3: Real Challenge
temperature, and availability of water and
nutrients. 23. Correct Option: (a)
 Net primary productivity (NPP) might
Explanation:
SC
positively respond to the carbon fertilization
effect. Although, evidence shows that  Statement 2 is incorrect: Mining dust
enhanced rates of photosynthesis in plants contains a variety of compounds hydrophobic
due to CO2 fertilization do not directly as well as hydrophilic depending on its
enhance all plant growth, and thus carbon source. The hydrophilic components leach
storage. into the soil with water very fast and
can reach up to the groundwater. While
Seed Stratification the leaching process for the hydrophobic
Stratification is a process of pre-treating components tends to be slower they too find
GS


seeds in order to simulate natural conditions their ways into the soil with time.
that seeds would experience in the soil over  Statement 3 is incorrect: Oil leaks can
winter. happen during the storage and transport of
Pre-treating seeds help the seed “break chemicals. This can be seen at most of the
CO

dormancy” and initiate the germination fuel stations. The chemicals present in the
process. fuel deteriorate the quality of soil and make
Stratification pre-treatments are applied them unsuitable for cultivation. These
S.


when the seed is being propagated in a chemicals can enter into the groundwater
greenhouse setting or in a way/schedule that through the soil and make the water
TE

is not aligned with the natural germination undrinkable.


cycle. Supplementary Notes
NO

 Seeds and seed mixes that are sown outdoors


in the fall do not require stratification pre-
Main Causes of Soil Pollution
treatment.  Soil pollution is a complex phenomenon,
and it can be triggered by a variety of things
DF

Effect of Ground Ozone of Vegetation and activities, from Industrial waste to the
and Crops excess use of chemical fertilizers. Every
The damage and injury that ground-level cause is linked with another. Pinpointing
.P


ozone (O3) causes vegetation has become at one particular cause is quite difficult.
increasingly evident over the past half- However, the leading causes are listed
W

century with a large body of observational below.


W

PPP-PTS-4341/092023/15 11
W
https://www.pdfnotes.co/

Industrial Activity 24. Correct Option: (d)


 Industrial activity has been the biggest
contributor to the problem in the last
Explanation:
century, especially since the amount of  Option (d) is correct
mining and manufacturing has increased.
Most industries are dependent on extracting Supplementary Notes
minerals from the Earth.
Extra- Terrestrial causes of Climate
 Whether it is iron ore or coal, the by- Change
products are contaminated, and they are
not disposed of in a manner that cannot be  Solar Irradiance: There is a regular
considered safe. As a result, the industrial fluctuation in the amount of energy
waste lingers in the soil surface for a long irradiated from the outer surface of the
time and makes it unsuitable for use. sun or the photosphere, this alteration
brings changes in the temperatures and
Agricultural Activities precipitation. The changes in the net
 The utilization of chemicals has gone up amount of solar energy received on earth
tremendously since technology provided us may also change because of change in
with modern pesticides and fertilizers. They relative distance between earth and sun.
are full of chemicals that are not produced
in nature and cannot be broken down by it.  Sunspots: These are dark and cooler patches
As a result, they seep into the ground after on the sun which increase and decrease in

RE
they mix with water and slowly reduce the a cyclical manner. When the number of
fertility of the soil. sunspots increases, cooler and wetter
 Other chemicals damage the composition weather and greater storminess
of the soil and make it easier to erode by occur. A decrease in sunspot numbers
water and air. Plants absorb many of these is associated with warm and drier
pesticides, and when they decompose, they conditions.
cause soil pollution since they become a part  Millankovitch Oscillations: This theory
of the land.
infers cycles in the variations in the earth’s
O
Waste Disposal orbital characteristics around the sun, the
 Finally, a growing cause for concern is how wobbling of the earth and the changes in the
we dispose of our waste. While industrial earth’s axial tilt. All these alter the amount
waste is sure to cause contamination, there of insolation received from the sun, which in
SC
is another way in which we are adding to the turn, might have a bearing on the climate.
pollution. Every human produces a certain  Orbital eccentricity of the earth: The
amount of personal waste products by way eccentricity of the earth orbit is between
of urine and faeces.
0.001 & 0.054. Earth takes 95,000 years
 While much of it moves into the sewer to achieve its maximum eccentricity from
system, there is also a large amount that minimum. As eccentricity increases, days
is dumped directly into landfills in the of summer increase both in northern and
form of diapers. Even the sewer system southern hemisphere.
ends at the landfill, where the biological
GS

waste pollutes the soil and water. This is  Obliquity or the axial tilt of Earth’s
because our bodies are full of toxins and rotational axis: The axial tilt or obliquity of
chemicals which are now seeping into earth’s rotational axis is the angle between
the land and causing pollution of soil. the axis of revolution and the axis of rotation
CO
Accidental Oil Spills of the Earth. The significance of obliquity is
that it controls the latitudinal distribution
 Oil leaks can happen during the storage of solar radiant energy and intensity and
and transport of chemicals. This can be seen duration of different seasons. The change
S.

at most of the fuel stations. The chemicals


in the obliquity is directly related to the
present in the fuel deteriorate the quality
temperature difference between summers
TE

of soil and make them unsuitable for


cultivation. These chemicals can enter into and winters. Smaller is the obliquity;
the groundwater through the soil and make smaller is the change in temperature
NO

the water undrinkable. between summers and winters.

Acid Rain  Axial precision or Precision of


equinoxes: It is slow and continuous
 Acid rain is caused when pollutants present
DF

change in the orientation of the rotational


in the air mix up with the rain and fall back
axis of the earth due to effects of gravity
on the ground. The polluted water could
dissolve away some of the essential nutrients of the sun, the moon and of the nearby
.P

found in soil and change the structure of the planets. Due to the effect of gravitational
soil. forces on earth the direction of axis of
rotation changes and it forms a complete
W

L3: Real Challenge


conical motion. This is also referred to as
W

12 PPP-PTS-4341/092023/15
W
https://www.pdfnotes.co/

wobbling of the rotational axis of the earth.  The saturation horizon of calcite (relatively
This cycle of axial precision is completed in less soluble mineral form found in the
every 26,000 year. The length and warmth shells of planktonic algae, some corals,
of the season alter due to axial precision. It echinoderms, and some mollusks) occurs at a
is seen that due to axial precision winters in greater ocean depth than that for aragonite
northern hemisphere become much longer (more soluble form of calcium carbonate; it
and warmer while summers in southern is found in most corals, most mollusks)
hemisphere becomes much longer and less
 The current increased rate of
warm.
dissolution of atmospheric CO2 into the
L3: Real Challenge ocean results in an imbalance in the
carbonate compensation depth (CCD),
the depth at which all carbonate is
25. Correct Option: (b) dissolved.
Explanation:  As the pH of the ocean falls, it results
 Option (b) is correct in a shallowing of the CCD, thus
exposing more of the shells trapped
Supplementary Notes in the sediments to under saturated
Ocean Acidification conditions causing them to dissolve,
which will help buffer ocean acidification
 Increase in CO2 concentrations not only but over a long time scale of a thousand

RE
leads to warmer oceans but also to more years.
acidic oceans.
L2: Approachable
 As the uptake of atmospheric carbon dioxide
by the ocean increases, the concentration of
hydrogen ions in the ocean increases, the 26. Correct Option: (d)
concentration of carbonate ions decreases,
the pH of the oceans decreases and the Explanation:

known as ocean acidification.


O
oceans become less alkaline – this process is  All statements are correct
Supplementary Notes
 Carbonic acid reacts with carbonate
ions in the water to form bicarbonates. UN Environment Programme
SC
Ocean Acidification will convert more
 In the recently concluded UN Environment
carbonate ions (which are required for
Assembly, 175 countries supported a
shell-building by marine organisms)
landmark resolution to end plastic waste by
into bicarbonates, the animals need
forming an intergovernmental committee
to expend more energy to build their
to negotiate and finalize a legally binding
shells. As a result, the shells end up
agreement by 2024.
being thinner and more fragile.
 The high-level event, named UNEP@50, will
 In the long run, this reaction will allow
mark the 50th anniversary of the United
GS

the ocean to soak up excess carbon dioxide


because more acidic water will dissolve Nations Environment Programme which
more rock, release more carbonate ions, was founded in 1972.
and increase the ocean’s capacity to absorb  Under UNEP@50, a year-long series
carbon dioxide of activities and outreach events are
CO
 Deep, cold ocean waters are naturally under being organized to mark UNEP’s 50th
saturated with carbonate ions causing anniversary. These events are being held
the shells of most calcifying organisms to to recognize the progress made on global
S.

dissolve. environmental matters and address the


upcoming planetary challenges.
TE

 Surface waters are over saturated with


carbonate ions and do not readily dissolve  The United Nations Environment
shells of calcifying organisms. Programme (UNEP) is responsible for
NO

coordinating responses to environmental


 The saturation horizon is the level below
issues within the United Nations system.
which calcium carbonate minerals undergo
dissolution.  As a member of the United Nations
DF

Development Group, UNEP aims to help the


 Ocean acidification causes this horizon
world meet the 17 Sustainable Development
to rise vertically in the water column so
Goals.
more and more calcifying organisms will be
.P

exposed to under saturated water and thus  UNEP hosts the secretariats of several
vulnerable to dissolution of their shells and multilateral environmental agreements and
W

skeletons. research bodies, including:


W

PPP-PTS-4341/092023/15 13
W
https://www.pdfnotes.co/

 Minamata Convention Reasons for this are:


 United Nations Convention on Biological  Change in Albedo: Albedo is a measure of
Diversity how much light that hits a surface is reflected
without being absorbed. When bright and
 Convention on International Trade in
reflective ice (with more albedo) melts,
Endangered Species of Wild Fauna and
it gives way to a darker ocean (lowering
Flora (CITES)
albedo); this amplifies the warming trend
 Basel Convention because the ocean surface absorbs more
 Stockholm Convention heat from the Sun than the surface of snow
and ice.
 Rotterdam Convention
 Changing Ocean currents: Ocean
 Montreal Protocol currents normally bring in warmer water
 Vienna Convention from the Pacific, and colder water exits out
of the Arctic into the Atlantic. But those
 Convention on Migratory Species currents may be changing because more
 Tehran Convention melting ice is injecting the Arctic Ocean with
freshwater. The missing ice also exposes the
 Bamako Convention
surface waters to more wind. This mixes
 Carpathian Convention up colder freshwater at the surface and
Climate & Clean Air Coalition (CCAC) warmer saltwater below, raising surface

RE

temperatures and further melting ice.
 In 1988, the World Meteorological
Organization and UNEP established  Changing Weather: Ocean currents
drive the powerful polar jet stream, which
the Intergovernmental Panel on Climate
moves hot and cold air masses around the
Change (IPCC). UNEP is also one of
Northern Hemisphere. This is a product of
several Implementing Agencies for the
the temperature differences between the
Global Environment Facility (GEF) and the
Arctic and the tropics. But as the Arctic
O
Multilateral Fund for the Implementation warms, the jet stream now undulates wildly
of the Montreal Protocol north and south. This has been injecting the
 The following reports are published by Arctic with warm air. Thunderstorms are
UNEP: also much more likely to occur in the tropics
than the higher latitudes. The storms
SC
 Actions on Air Quality
transport heat from the surface to higher
 Emissions Gap Report levels of the atmosphere, where global wind
 Global Environment Outlook patterns sweep it toward higher latitudes.
The abundance of thunderstorms in tropics
 The Rise of Environmental Crime creates a near-constant flow of heat away
 Frontiers 2022: Noise, Blazes and from the tropics towards the Arctic
Mismatches  There is no Antarctic amplification.
Antarctic warming has been similar to
GS

L3: Real Challenge


the global average, although some parts
are warming much faster. This is because,
27. Correct Option: (d) Antarctica is surrounded by the vast
Southern Ocean, which is soaking up much
CO
Explanation: of the atmosphere’s excess heat.
 All statements are correct L3: Real Challenge
Supplementary Notes
S.

Arctic Amplification 28. Correct Option: (c)


TE

 Over the past 30 years, the Arctic has Explanation:


warmed at roughly twice the rate as the  Option (c) is correct
NO

entire globe, a phenomenon known as Arctic


amplification. Supplementary Notes
 This means that global warming and climate Internal Carbon Price
DF

change are impacting the Arctic more than  An internal carbon price is a value that
the rest of the world. companies voluntarily set for themselves,
Global temperatures from 2000–2009 were in order to internalise the economic cost
.P


on average about 0.6°C higher than they of their greenhouse gas emissions. It can
were from 1951–1980. The Arctic, however, be used both as risk management tool and
W

was about 2°C warmer. as part of a company’s decarbonisation


W

14 PPP-PTS-4341/092023/15
W
https://www.pdfnotes.co/

strategy. An internal carbon price can help 30. Correct Option: (a)
companies enhance their global strategies to
become more resilient to regulatory climate Explanation:
policies and more favourable to emission  Option (a) is correct
reductions.
Supplementary Notes
Internal carbon pricing primarily takes
two forms: Environmental Impact Assessment (EIA)
 A shadow price: which represents a carbon  It started in 1976-77 when the Planning
value (determined by the company) that is Commission asked the Department of
incorporated into investment decisions and Science and Technology to examine the
applied to the greenhouse gas emissions river-valley projects from an environmental
generated by projects; angle.

 An internal carbon tax: a levy that  Till 1994, environmental clearance from the
companies voluntarily apply to their Central Government was an administrative
operations and that increases operating decision and lacked legislative support.
costs depending on the resulting greenhouse  On 27 January 1994, the then Union
gas emissions; the company then uses the Ministry of Environment and Forests,
proceeds of this tax as it sees fit. Internal under the Environmental (Protection)
Carbon price is an internationally recognised Act 1986, promulgated an EIA notification

RE
business tool that enables companies to making Environmental Clearance (EC)
create resources which are invested in low mandatory for expansion or modernization
carbon technologies, which help reduce of any activity or for setting up new projects
future emissions and lower operating costs. listed in Schedule 1 of the notification.
L3: Real Challenge  The Ministry of Environment, Forests and
Climate Change (MoEFCC) notified new
EIA legislation in September 2006.
29. Correct Option: (c)
O  According to Convention of Biological
Explanation: Diversity, Environmental Impact
Assessment (EIA) is a process of evaluating
 Option (c) is correct
the likely environmental impacts of a
SC
Supplementary Notes proposed project or development, taking
into account inter-related socio-economic,
Global Dimming cultural and human-health impacts, both
 Along with the phenomenon of global beneficial and adverse.
warming, global dimming is also a  United Nations environment Programme
consequence of climate change and changing (UNEP) defines Environmental Impact
atmospheric phenomenon. Assessment (EIA) as a tool used to identify
An English scientist, Gerald Stanhill, during the environmental, social and economic
GS


1985, first noticed a 22% drop in sunlight in impacts of a project prior to decision-
Israel as compared to his findings in 1950. making. It aims to predict environmental
He termed it as global dimming. impacts at an early stage in project
planning and design, find ways and means
CO
 It is the drop in the amount of average to reduce adverse impacts, shape projects to
solar energy reaching the Earth’s suit the local environment and present the
surface due to shielding caused by predictions and options to decision-makers.
suspended air particulate materials.
S.

 By using EIA both environmental and


 Air polluting particles seeded water vapour economic benefits can be achieved, such
TE

in the cloud are the major cause behind as reduced cost and time of project
global dimming. implementation and design, avoided
NO

 Global dimming can have some very treatment/clean-up costs, and impacts of
devastating impacts on us. laws and regulations.
 The drought in Sub-Saharan Africa during L2: Approachable
DF

1970s and 1980s is considered to be caused


by global dimming. Because, cloudy regions
can adversely affect crop ripening, water
31. Correct Option: (b)
.P

bodies, oxygen production and global Explanation:


rainfall patterns.
 Statement 1 is correct: The project is
W

L3: Real Challenge supported by the World Bank.


W

PPP-PTS-4341/092023/15 15
W
https://www.pdfnotes.co/

 Statement 2 is correct: Ministry of climate adaptation including policy inputs


Environment, Forest and Climate Change for scaling-up of the community-based
is the concerned ministry while National climate adaptation approach within the
Institute of Rural Development and NRLM.
Panchayati Raj is the implementing agency  Project management and impact evaluation
of this project.
component will invest in: (i) establishment
 Statement 3 is incorrect: The objective of climate adaptation units staffed with
of SLACC is to improve adaptive capacity full-time professionals within the NRLM
of rural poor engaged in farm-based and the State Rural Livelihoods Mission
livelihood. (SRLMs) of the participating states; (ii)
Supplementary Notes establishment of a monitoring system and
evaluation arrangements (baseline, mid-
Sustainable Livelihoods and Adaptation term and end-of-term).
to Climate Change L3: Real Challenge
 India signed the Grant Agreement with
World Bank for Special Climate Change
fund Assistance for Sustainable Livelihoods 32. Correct Option: (c)
and Adaptation to Climate Change Project
Explanation:
in 2019.
Both statements are correct

RE

 Ministry of Environment, Forest and Climate
Change is the concerned ministry while Supplementary Notes
National Institute of Rural Development
and Panchayati Raj is the implementing Carbon emission in Agriculture
agency.  The global food system is responsible for
 The objective of the Sustainable Livelihoods ~21–37% of annual emissions, as commonly
and Adaptation to Climate Change Project reported using the 100-year Global Warming
Potential (more on this later).
O
for India is to improve adaptive capacity
of the rural poor engaged in farm-based  The composition of gases emitted by the
livelihoods to cope with climate variability food system does not reflect the overall
and change.
global emissions balance, however, with
The project has 3 components: agricultural activity generating around half
SC
of all anthropogenic methane emissions and
 Planning, service provision and
around three-quarters of anthropogenic
implementation of climate change
N2O.
adaptation component will support risk
assessment, planning, service provision  The non-CO2 gases methane and nitrous
and implementation of climate adaptation oxide comprise a uniquely large share of
interventions. The key activities include: agricultural emissions.
(I) community-led risk assessment and
 Animals are the source of atmospheric
GS

participatory planning of climate adaptation


carbon on earth. Rearing animals at large
interventions; (ii) provision of strategic
scale tilts agriculture’s carbon budget more
climate change adaptation services through
towards being source. Soil organic material
knowledge assimilation and partnerships
with resource institutions; and (iii) is one of the major sinks for the carbon.
CO
implementation of climate adaptation Agricultural practices like ploughing and
interventions in agriculture by community flood irrigation disturb the soil frequently,
institutions (self-help groups/federations) causing its rapid release.
S.

utilizing the Community Climate Adaptation L2: Approachable


(CCA) grants upon approval of a community
TE

adaptation plan.
33. Correct Option: (b)
 Scaling and mainstreaming community-
NO

based climate adaptation component will Explanation:


enable support and build capacity for the
 Option (b) is correct
implementation of climate adaptation
interventions, and to develop the strategy Supplementary Notes
DF

for scaling up. Key activities include:


(I) capacity building of National Rural Geoengineering
Livelihoods Mission (NRLM) national  Any engineering intervention that is
.P

and state staff and creation of a cadre of implemented/proposed to mitigate/


Community Resource Persons (CRPs); (ii) accelerate climate change at global scale
W

building knowledge support system for will fall under geoengineering.


W

16 PPP-PTS-4341/092023/15
W
https://www.pdfnotes.co/

 There is wide range of proposed  Ocean Alkalinity Enhancement:


geoengineering techniques. Generally, these Grinding up, dispersing, and dissolving
can be grouped into two categories: rocks such as limestone, silicates, or calcium
hydroxide in the ocean to increase its ability
Solar Radiation Management (SRM) or to store carbon and directly ameliorate
Solar Geoengineering ocean acidification.
 SRM techniques aim to reflect a small Rooftop solar-energy aims to fulfil the
proportion of the Sun’s energy back into energy requirement at a local scale, it is not
space, counteracting the temperature rise considered as geoengineering technique.
caused by increased levels of greenhouse
gases in the atmosphere which absorb L1 : Elementary
energy and raise temperatures.
Some proposed techniques include: 34. Correct Option: (a)
 Albedo enhancement: Increasing the Explanation:
reflectiveness of clouds or the land surface  Option (a) is correct
so that more of the Sun’s heat is reflected
back into space. Supplementary Notes
 Space reflectors: Blocking a small Stratospheric ozone layer depletion
proportion of sunlight before it reaches the  The term “ozone depletion” means more

RE
Earth. than just the natural destruction of ozone;
 Stratospheric aerosols: Introducing it means that ozone loss is exceeding ozone
creation.
small, reflective particles into the upper
atmosphere to reflect some sunlight before  Putting additional ozone-destroying
it reaches the surface of the Earth. compounds such as CFCs into the
atmosphere is like increasing the size of the
Greenhouse Gas Removal (GGR) or holes in our “bucket” of ozone. The larger
holes cause ozone to leak out at a faster rate
Carbon Geoengineering

O
GGR techniques aim to remove carbon
than ozone is being created. Consequently,
the level of ozone protecting us from
dioxide or other greenhouse gases from ultraviolet radiation decreases.
the atmosphere, directly countering the
increased greenhouse effect and ocean  During the last 15 years, an additional
SC
mechanism was found in the areas over the
acidification. These techniques would have
Antarctic and Arctic that rapidly destroys
to be implemented on a global scale to have ozone. Over the Earth’s poles during
a significant impact on greenhouse gas their respective winters, the stratosphere
levels in the atmosphere. cools to very cold temperatures and polar
stratospheric clouds (PSCs) form.
Some proposed techniques include:
 In the polar stratosphere, nearly all of
 Afforestation: Engaging in a global-scale
the chlorine is in the form of inactive or
tree planting effort. “reservoir” gases such as hydrogen chloride
GS

 Biochar: ‘Charring’ biomass and burying it (HCl) and chlorine nitrate (ClONO2) that
so that its carbon is locked up in the soil. do not react with ozone or each other.
However, chemical reactions of these
 Bio-energy with carbon capture and “reservoir” chlorine gases can occur on the
CO
sequestration: Growing biomass, burning polar stratospheric cloud particle surfaces,
it to create energy and capturing and converting the chlorine gases into very
sequestering the carbon dioxide created in reactive forms that rapidly destroy ozone.
the process.
S.

 This “polar chemistry” on the stratospheric


 Ambient Air Capture: Building large cloud particles has caused very large
TE

machines that can remove carbon dioxide decreases in ozone concentrations over
directly from ambient air and store it Antarctica and the Arctic. In fact, ozone
elsewhere. levels drop so low in spring over Antarctica
NO

that scientists describe this loss as the


 Ocean fertilization: Adding nutrients to “Antarctic Ozone Hole.”
the ocean in selected locations to increase
 CO2 affects temperatures in both the
primary production which draws down
troposphere and the stratosphere
DF

carbon dioxide from the atmosphere. and formation of O3 is an exothermic


 Enhanced Weathering: Exposing large process. CH4 affects the levels of
quantities of minerals that will react with reactive hydrogen oxides in the
.P

carbon dioxide in the atmosphere and troposphere and stratosphere that can
storing the resulting compound in the ocean react with ozone.
W

or soil. L2: Approachable


W

PPP-PTS-4341/092023/15 17
W
https://www.pdfnotes.co/

35. Correct Option: (a)  Recently, UNFCCC has joined hands


with the Air Carbon Exchange (ACX), the
Explanation: world’s first fully digital carbon exchange,
 Statement 2 is incorrect: Gold Standard to promote carbon offsetting via emission
Voluntary Emission Reductions (VER) is a reductions.
certificate issued by World Wide Fund for  The ACX, the Singapore-based
Nature (WWF). exchange, a central marketplace for
Supplementary Notes carbon credits, is the first exchange
in Asia to list CERs held in the CDM
Carbon Offsetting registry - the database that holds
 Carbon Offsetting is the prevention or and tracks CERs, including their
removal of Greenhouse Gases (more retirement.
commonly referred to as Carbon Emissions) L2: Approachable
from the atmosphere, to compensate for
emissions created elsewhere.
 Carbon Offsetting works in practice via
36. Correct Option: (b)
projects around the world that prevent Explanation:
or remove Greenhouse Gases, such as
 Statement 1 is incorrect: The yields of
tree planting, deforestation prevention/

RE
major cereals crops especially like wheat is
protection, and renewable projects such as
likely to be reduced due to decrease in crop
Wind, Solar and Hydro Power.
growth duration, increased respiration,
 Third party standards ensure the project and /or reduction in rainfall/irrigation
is “additional”, meaning the project would water supplies due to rise in atmospheric
not have otherwise occurred if it was not temperature.
funded by the Carbon Credit purchases
themselves.  Statement 2 is correct: Increase in
O
temperature can affect the survival and
 In addition, the standards also ensure distributions of pest populations and thus
the projects have secondary benefits (not developing new equilibrium between crops
just offsetting emissions) – providing and pests.
employment, education, and a positive
SC
impact on the biodiversity and wildlife  Statement 3 is correct: Increase in
within the region. atmospheric carbon dioxide has a fertilization
effect on crops with C3 photosynthetic
 There are many different third-
pathway and thus promotes their growth
party standards, but the three
and productivity.
largest and most regulated are the
Verified Carbon Standard (VCS), Supplementary Notes
Gold Standard Voluntary Emission
Reductions (VER), and the United Impact of climate change on agriculture
GS

Nations Certified Emission Reductions  Increase in ambient CO2 is beneficial since


(CER) programmes. In particular, CER this leads to increased photosynthesis in
projects are regulated by the United several crops, especially crops with C3
Nations directly. mechanism of photosynthesis such as wheat
CO
 Carbon markets facilitate the trading of and rice, and decreased evaporative losses.
emission reductions. A carbon market  Despite this, the yields of major cereals crops
existed under Kyoto Protocol but is no especially like wheat is likely to be reduced
S.

longer there because the Protocol itself due to decrease in crop growth duration,
expired last year. increased respiration, and /or reduction in
TE

 The developing countries wanted their rainfall/irrigation water supplies due to rise
unused carbon credits to be transitioned to in atmospheric temperature.
NO

the new market.  Increase in temperature can reduce crop


 The Glasgow Pact (COP 26) allowed these duration, increase crop respiration rates,
carbon credits to be used in meeting alter photosynthesis process, affect the
DF

countries’ first NDC targets. These cannot survival and distributions of pest populations
be used for meeting targets in subsequent and thus developing new equilibrium
NDCs. That means, if a developed country between crops and pests, hastens nutrient
.P

wants to buy these credits to meet its own mineralization in soils, decrease fertilizer
emission reduction targets, it can do so till use efficiencies, and increase in evapo-
W

2025. transpiration.
W

18 PPP-PTS-4341/092023/15
W
https://www.pdfnotes.co/

 Climate change can affect agriculture  Biorock is the name given to the substance
through their direct and indirect effects formed by electro accumulation of
on the crops, soils, livestock and pests. minerals dissolved in seawater on steel
Increase in atmospheric carbon dioxide structures that are lowered onto the sea
has a fertilization effect on crops with C3 bed and are connected to a power source,
photosynthetic pathway and thus promotes in this case solar panels that float on the
their growth and productivity. surface.
L3: Real Challenge  The M.S. Swaminathan Research Foundation
(MSSRF) and the Tamil Nadu Fisheries
Development Corporation (TNFDC) have
37. Correct Option: (d) been installing artificial reefs to help fishers
in some of the villages in Tamil Nadu.
Explanation:
L2: Approachable
 Option (d) is correct
Supplementary Notes 38. Correct Option: (d)
Artificial Reef Explanation:
 Artificial reefs are intentionally placed  Statement 2 is incorrect: UN REDD+ has
benthic structures built of natural or man- set the target of reduction of 1 gigaton per
made materials, which are designed to year from 2025.
protect, enhance, or restore components

RE
of marine ecosystems. Supplementary Notes
 Their ecological structure and function, UN REDD+
vertical relief, and irregular surfaces vary  The United Nations Collaborative
according to location, construction, and Programme on Reducing Emissions from
degree to which they mimic natural habitats, Deforestation and Forest Degradation in
such as coral reefs. Developing Countries (UN-REDD) is the
An AR can serve several purposes – UN knowledge and advisory platform on

O
recreational activities (surfing, scuba the forest solutions to the climate crisis.
diving, snorkelling, and tourism), disaster  Its goal is to help realize forest solutions
management, coastal protection, to the climate emergency by avoiding
managing and promoting marine carbon emissions and fostering carbon
SC
biodiversity, increasing fish catch, and sequestration.
preventing trawling.  In pursuing this international goal, UN-
 Most artificial reefs are purposefully REDD promotes approaches that ensure the
designed and deployed using long-lasting environmental integrity of carbon emissions
and non-toxic materials that provide a high reductions while supporting non-carbon
degree of surface and structural diversity. benefits – from safeguarding biodiversity to
supporting local livelihoods and promoting
 Today, we generally recognize several the rights of indigenous peoples.
main goals for artificial reefs:
UN-REDD was launched in 2008 and builds
GS


 Replace structure and habitat diversity on the convening capacity and technical
in places where it has been lost (due expertise of the Food and Agriculture
to dredging, development, storms, Organization of the United Nations
bleaching, etc.) (FAO), the United Nations Development
CO
 Increase the size of reefs or available Programme (UNDP) and the United Nations
structure to enhance local marine Environment Programme (UNEP).
resources and improve biodiversity  UN REDD+ help 65 partner countries to
S.

 Create artificial dive/snorkelling sites advance the implementation of the Paris


to relieve tourism pressure on natural Agreement, particularly Articles 5 and 6,
in order to reduce deforestation, promote
TE

reefs
sustainable land uses, advance international
 Create attractive or art inspired reefs to cooperative approaches to climate mitigation
increase awareness and communicate
NO

and mobilize climate finance to turn the tide


reef issues to the general public on tropical deforestation.
 It is important to recognize that artificial  UN-REDD supports nationally led REDD+
reefs only work in areas where water quality processes and promotes the informed
DF

is still conducive to coral growth. and meaningful involvement (FPIC) of


all stakeholders, including indigenous
 In India, Zoological Survey of India (ZSI),
peoples and local communities, in the
with help from Gujarat’s forest department,
.P

implementation of REDD+ activities agreed


is attempting for the first time a process to
under the UNFCCC.
restore coral reefs using biorock or mineral
W

accretion technology. L2: Approachable


W

PPP-PTS-4341/092023/15 19
W
https://www.pdfnotes.co/

39. Correct Option: (a) 40. Correct Option: (c)


Explanation: Explanation:
 Statement 2 is incorrect: Many  Both statements are correct
fluorinated gases have very high global
warming potentials (GWPs) relative to Supplementary Notes
other greenhouse gases. Ground level or “bad” ozone
Supplementary Notes  Ozone (O3) is a gas that occurs both in the
Earth’s upper atmosphere and at ground
Global Warming Potential (GWP) level. Ozone can be “good” or “bad” for the
 Global warming potential is the heat health and the environment, depending on
absorbed by any greenhouse gas in the its location in the atmosphere.
atmosphere, as a multiple of the heat that  Ground level or “bad” ozone is an air
would be absorbed by the same mass of pollutant that is harmful to breathe and it
carbon dioxide. damages crops, trees and other vegetation.
 The Global Warming Potential (GWP) It is a main ingredient of urban smog.
was developed to allow comparisons of the  Ground-level ozone is not emitted directly
global warming impacts of different gases. into the air, but is created by chemical
Specifically, it is a measure of how much reactions between oxides of nitrogen (NOx)

RE
energy the emissions of 1 ton of a gas will and volatile organic compounds (VOC) in
absorb over a given period of time, relative the presence of sunlight. Emissions from
to the emissions of 1 ton of carbon dioxide industrial facilities and electric utilities,
(CO2). motor vehicle exhaust, gasoline vapors, and
 The larger the GWP, the more that a given chemical solvents are some of the major
gas warms the Earth compared to CO2 over sources of NOx and VOC.
that time period. The time period usually L2: Approachable
O
used for GWPs is 100 years.
 Carbon dioxide (CO2) has a GWP of 1 and
serves as a baseline for other GWP values. 41. Correct Option: (c)
• The larger the GWP, the more warming Explanation:
the gas causes.
SC
 All pairs are correctly matched
 Many fluorinated gases have very high
global warming potentials (GWPs) Supplementary Notes
relative to other greenhouse gases.
Greenhouse Gas Removal
Fluorinated gases are well-mixed in the
atmosphere, spreading around the world (GGR) or Carbon Geoengineering
after they’re emitted.  GGR techniques aim to remove carbon
 For example, methane’s 100-year GWP is dioxide or other greenhouse gases from
the atmosphere, directly countering the
GS

21, which means that methane will cause


21 times as much warming as an equivalent increased greenhouse effect and ocean
mass of carbon dioxide over a 100- year time acidification. These techniques would have
period. Methane (CH4) has a GWP more to be implemented on a global scale to have
than 20 times higher than CO2 for a a significant impact on greenhouse gas
CO
100-year time scale. levels in the atmosphere. Some proposed
techniques include:
 Alternate metric is the Global Temperature
Potential (GTP). While the GWP is a  Afforestation. Engaging in a global-
S.

measure of the heat absorbed over a scale tree planting effort.


Biochar. ‘Charring’ biomass and
TE

given time period due to emissions 


of a gas, the GTP is a measure of the burying it so that its carbon is locked up
temperature change at the end of that in the soil.
NO

time period (again, relative to CO2).  Bio-energy with carbon capture


 The calculation of the GTP is more and sequestration. Growing biomass,
complicated than that for the GWP, as it burning it to create energy and capturing
DF

requires modelling how much the climate and sequestering the carbon dioxide
system responds to increased concentrations created in the process.
of GHGs (the climate sensitivity) and how  Ambient Air Capture. Building large
.P

quickly the system responds (based in part machines that can remove carbon dioxide
on how the ocean absorbs heat). directly from ambient air and store it
W

L3: Real Challenge elsewhere.


W

20 PPP-PTS-4341/092023/15
W
https://www.pdfnotes.co/

 Ocean Fertilisation. Adding nutrients  The government has set up a budget


to the ocean in selected locations to provision of Rs.350 crores for the year
increase primary production which 2015-16 and 2016-17, with an estimated
draws down carbon dioxide from the requirement of Rs. 181.5 crores for the
atmosphere. financial year 2017-18 for NAFCC.
 Enhanced Weathering. Exposing large  The projects under NAFCC prioritize the
quantities of minerals that will react needs that build climate resilience in the
with carbon dioxide in the atmosphere areas identified under the SAPCC (State
and storing the resulting compound in Action Plan on Climate Change) and the
relevant Missions under NAPCC (National
the ocean or soil.
Action Plan on Climate Change).
 Ocean Alkalinity Enhancement.
 Considering the existing arrangement with
Grinding up, dispersing, and dissolving
NABARD as National Implementing Entity
rocks such as limestone, silicates, (NIE) for Adaptation Fund (AF) under
or calcium hydroxide in the ocean to Kyoto Protocol and its presence across the
increase its ability to store carbon and country, NABARD has been designated
directly ameliorate ocean acidification. as National Implementing Entity (NIE)
L3: Real Challenge for implementation of adaptation projects
under NAFCC by Govt. of India.
 Under this arrangement, NABARD would
42. Correct Option: (d)

RE
perform roles in facilitating identification
Explanation: of project ideas/concepts from State
Action Plan for Climate Change (SAPCC),
 Statement 1 is incorrect: The funding for project formulation, appraisal, sanction,
the National Clean Energy Fund is raised disbursement of fund, monitoring &
through carbon tax on the use of coal by evaluation, and capacity building of
industries. stakeholders including State Governments.
 Statement 2 is incorrect: National L2: Approachable
Adaptation Fund (NAF) and National
O
Adaptation Fund for Climate Change
(NAFCC) are two separate funds established 43. Correct Option: (d)
in the year 2014 and 2015 respectively. Explanation:
SC
Supplementary Notes  Both statements are correct

National Clean Energy Fund Supplementary Notes


 The Fund was created to promote clean Coastal Fisheries Initiative - Challenge
energy, funded through an initial carbon Fund (CFI-CF)
tax on the use of coal by industries.  The World Bank Coastal Fisheries
 Governed by an Inter-Ministerial Group with Initiative – Challenge Fund (CFI-CF)
the Finance Secretary as the Chairman. recently launched a competition to seek
GS

collaborative solutions to reduce overfishing


 Its mandate is to fund research and
by supporting coordination among fishers
development of innovative clean energy and collaboration across seafood value
technology in the fossil and non-fossil fuel- chains.
based sectors.
CO
 The CFI-CF Global Knowledge Competition
National Adaptation Fund seeks innovative solutions that promote
the productive and sustainable use and
 The fund was established in July 2014 with
management of coastal fish stocks in Cabo
S.

a corpus of Rs. 100 crore to bridge the gap


Verde, Ecuador, Indonesia, and Peru.
between the need and the available funds.
TE

 The Coastal Fisheries Initiative (CFI) is


 The fund is operated under the Ministry of a collaborative, global effort funded by
Environment, Forests, and Climate Change the Global Environment Facility (GEF) to
NO

(MoEF&CC). preserve marine resources and ensure that


National Adaptation Fund for Climate coastal fisheries can continue to play their
crucial role in society, contributing to food
Change (NAFCC)
security, as well as economic and social
DF

 The National Adaptation Fund for Climate development.


Change (NAFCC) was established in August  The Challenge Fund, a child project of the
2015 to meet the cost of adaptation to climate CFI Program led by the World Bank, aims
.P

change for the State and Union Territories to engage businesses and the financing
of India that are particularly vulnerable to community in support of more productive
W

the adverse effects of climate change. and sustainable coastal fisheries.


W

PPP-PTS-4341/092023/15 21
W
https://www.pdfnotes.co/

 The Challenge Fund helps to make the 46. Correct Option: (a)
connection between fishing communities,
businesses, and investors, as well as the Explanation:
government.  Statement 1 is incorrect: At COP26, over
140 world leaders committed to “halt and
 The Challenge Fund focuses on concrete
reverse forest loss and land degradation
business ideas and supports coalitions
by 2030, whilst delivering sustainable
in finding innovative, long-term
development and promoting an inclusive
solutions to reach their common goals rural transformation.” The Forest & Climate
in designing responsible investment Leaders’ Partnership was launched at the
packages and identifying obstacles to their Forests & Climate Leaders’ Partnership
implementation. Summit on Monday 7th November 2022 at
L2: Approachable UNFCCC COP27 in Sharm El Sheikh.
 Statement 2 is correct: At this event,
leaders publicly set out their vision for the
44. Correct Option: (b) new Partnership and how it will contribute
Explanation: to the Glasgow Leaders’ Declaration on
Forests and Land Use goal to halt and
 Option (b) is correct reverse forest loss and land degradation
by 2030 whilst delivering sustainable

RE
Supplementary Notes
development and promoting an inclusive
CRISP-M Tool rural transformation.
 Union Minister of Rural Development  Statement 3 is incorrect: The Forest
& Panchayati Raj recently launched & Climate Leaders’ Partnership (FCLP)
Climate Resilience Information System will work with existing initiatives and
and Planning (CRISP-M) tool for Mahatma organisations to deliver ambition in six
specific Action areas which underpin the
O
Gandhi NREGA Scheme.
commitments set out at COP26.
 The CRISP-M tool will help embed climate
information in the GIS-based planning Supplementary Notes
and implementation of Mahatma Gandhi Forest and Climate Leaders’ Partnership
SC
NREGS. (FCLP)
L1 : Elementary  In 2021, at UNFCCC COP26, over 140
leaders, accounting for more than 90%
of the world’s forests, committed to work
45. Correct Option: (c) together to halt and reverse forest loss and
Explanation: land degradation by 2030, whilst delivering
sustainable development and promoting
 Both statements are correct an inclusive rural transformation in the
GS

Glasgow Leaders’ Declaration on Forests


Supplementary Notes
and Land Use.
Wildfire  Supporting the GLD, an ambitious package
of initiatives was announced, which included
CO
 A wildfire is an unplanned fire that burns in
a natural area such as a forest, grassland, the Forests, Agriculture Commodity Trade
or prairie. Wildfires are often caused by (FACT) dialogue road map along with $19.2
billion of public and private finance to
S.

human activity or a natural phenomenon


support countries to deliver on the headline
such as lightning, and they can happen at
2030 political target. Delivery of the GLD
TE

any time or anywhere. In 50% of wildfires


would reduce emissions by 3.5Gt CO2/year
recorded, it is not known how they started.
by 2030, 10% of the overall mitigation action
NO

 The risk of wildfires increases in extremely needed to keep 1.5 alive.


dry conditions, such as drought, and  Action Area 1: International
during high winds. Wildfires can disrupt Collaboration On The Sustainable Land
transportation, communications, power and Use Economy And Supply Chains.
DF

gas services, and water supply. They also


 Action Area 2: Mobilising Public
lead to a deterioration of the air quality, and And Donor Finance To Support
.P

loss of property, crops, resources, animals Implementation.


and people.
 Action Area 3: Shifting The Private
W

L2: Approachable Finance System


W

22 PPP-PTS-4341/092023/15
W
https://www.pdfnotes.co/

 Action Area 4: Power Forest Guardians  Afforestation and Sustainable Agriculture:


 Action Area 5: Strengthening And Expanding forest cover and adopting climate-
Scaling Carbon Markets For Forests resilient agricultural practices to sequester
carbon and enhance food security.
 Action Area 6: Building International
Partnerships And Incentives To Preserve  Green Finance: Mobilizing financial
High-Integrity Forests resources for green projects and investments
through innovative financing mechanisms.
L3: Real Challenge
 Adaptation and Resilience: Developing
strategies to adapt to the impacts of climate
47. Correct Option: (c) change and enhance resilience in vulnerable
Explanation: regions.

 All statements are correct  India’s Long-Term Low-Carbon Development


Strategy reflects the country’s commitment
Supplementary Notes to addressing climate change while
ensuring a prosperous and equitable future
Low-Carbon Development Strategy
for its citizens. By embracing sustainable
 India’s Long-Term Low-Carbon Development development and green technologies, India
Strategy is a visionary roadmap designed aims to become a global leader in the fight
to address the dual challenge of economic against climate change while improving the
growth and environmental sustainability.

RE
quality of life for its people.
The strategy, unveiled by the Indian
government, outlines a comprehensive L3: Real Challenge
plan to transition towards a low-carbon
and sustainable future over the coming
decades. 48. Correct Option: (b)
 At its core, the strategy aims to significantly Explanation:
reduce India’s carbon emissions while Statement 1 is incorrect: The Climate
O
fostering economic growth and social
development. It recognizes the importance

Neutral Now Initiative is one of several
initiatives launched by the United Nations
of aligning economic prosperity with
Framework Convention on Climate Change
environmental protection, acknowledging
(UNFCCC) secretariat to increase climate
that India’s energy demand is bound to
SC
rise due to its growing population and action by engaging non-Party stakeholders
expanding economy. Therefore, the strategy like sub-national governments, companies,
seeks to reshape the energy landscape organizations, and individuals.
through a combination of renewable energy  Statement 2 is incorrect: The Climate
expansion, energy efficiency measures, and Neutral Now Initiative was launched in
the promotion of clean technologies. 2015 following a mandate to promote the
 India’s historical contribution to cumulative voluntary use of the Clean Development
global GHG emissions is therefore Mechanism (CDM).
GS

minuscule despite having a share of 17%


Supplementary Notes
of the world’s population. India’s per capita
annual emissions are about a third of the Climate Neutral Now Initiative
global average.
The CDM allows emission-reduction
CO

 Key elements of India’s Long-Term Low- projects in developing countries to earn
Carbon Development Strategy include: certified emission reduction (CER) credits,
 Renewable Energy: India plans to harness its each equivalent to one tonne of CO2.
S.

abundant solar, wind, and hydro resources


 These CERs can be traded and sold and used
to scale up renewable energy generation.
by industrialized countries to meet a part of
TE

Ambitious targets have been set to increase


their emission reduction targets under the
the share of renewable energy in the energy
Kyoto Protocol.
mix.
NO

 Energy Efficiency: Implementing energy-  Thus, the Climate Neutral Now Initiative
efficient technologies and practices across promotes the voluntary use Clean
various sectors, including industry, Development Mechanism (CDM), which is a
DF

transportation, and agriculture, is a priority carbon market mechanism.


to reduce energy consumption.  It encourages and supports organizations
 Sustainable Mobility: Promoting electric and other interested stakeholders to act
.P

vehicles and sustainable transportation now in order to achieve a climate-neutral


infrastructure to reduce emissions from the world by 2050, as enshrined in the Paris
W

transport sector. Agreement.


W

PPP-PTS-4341/092023/15 23
W
https://www.pdfnotes.co/

 The initiative is not a certification scheme support in understanding, developing, and


for its participants. It has no participation implementing ambitious zero-emission
fee, and in addition to that, a participant transportation policies and plans, and
can stop participating at any point in time. showcase leadership.
Climate neutrality is a three step process,
L3: Real Challenge
which requires individuals, companies and
governments to:
 Measure their climate footprint 50. Correct Option: (c)
 Reduce their emissions as much as Explanation:
possible
 All statements are correct
 Offset what they cannot reduce with UN
certified emission reductions. Supplementary Notes
 Over time the need for offsets is going to Ozone Deplition
decline as energy systems become ever  Phytoplankton form the foundation
more low carbon, healthy ecosystems of aquatic food webs. Phytoplankton
like forests expand and we arrive at a productivity is limited to the euphotic zone,
climate neutral world in the second half the upper layer of the water column in
of the century.
which there is sufficient sunlight to support
L3: Real Challenge net productivity. Exposure to solar UVB

RE
radiation has been shown to affect both
orientation and motility in phytoplankton,
49. Correct Option: (a) resulting in reduced survival rates for these
Explanation: organisms.
 Statement 2 is incorrect: It will work  Increases in UVB radiation could affect
towards all sales of new cars and vans being terrestrial and aquatic biogeochemical
O
zero emission globally by 2040 and by no cycles, thus altering both sources and sinks
later than 2035 in leading markets. of greenhouse and chemically important
trace gases (e.g., carbon dioxide, carbon
 Statement 3 is incorrect: It builds off the
monoxide, carbonyl sulfide, ozone, and
momentous foundation of the ‘Zero Emission
possibly other gases). These potential
Vehicles Declaration’ (ZEV Declaration)
SC
changes would contribute to biosphere-
generated at COP26 and hosted by the UK
COP Presidency in collaboration with the atmosphere feedbacks that mitigate or
High-Level UN Climate Champions and the amplify the atmospheric concentrations of
Climate Group. these gases.
 UVB radiation affects the physiological and
Supplementary Notes developmental processes of plants. Despite
Accelerating to Zero coalition (A2Z) mechanisms to reduce or repair these effects
and an ability to adapt to increased levels of
GS

 Accelerating to Zero (A2Z) Coalition was


UVB, plant growth can be directly affected
launched at the conference of parties
by UVB radiation.
(COP27) as the next step to secure more
ambitious commitments to zero-emission  Ozone layer depletion increases the
vehicles transition aligned with the Paris amount of UVB that reaches the Earth’s
CO
Agreement. The A2Z Coalition connects surface. Laboratory and epidemiological
the world’s leading organizations on zero- studies demonstrate that UVB causes non-
emission transportation, creating a platform melanoma skin cancer and plays a major
S.

to support understanding, developing, and role in malignant melanoma development.


implementing ambitious zero-emission. In addition, UVB has been linked to the
TE

 The A2Z Coalition connects the world’s development of cataracts, a clouding of the
leading organizations on zero-emission eye’s lens.
NO

transportation, creating a platform to L3: Real Challenge


DF

™™™™™
.P
W
W

24 PPP-PTS-4341/092023/15
W

You might also like